You are on page 1of 60

TORTS

OUTLINE

Chapter 1: Intentional Torts- Interference with Persons and Property
A.)

Intent
Restatements
Not statutes, they consist of black letter rules
First = 1939
Second = 1964
Third = 1992 (revising)
3 Definitions for Intent (liable if you are one of them)
a) Desire to harm or offend, or a apprehension thereof to a particular person
b) Knowledge to a substantial certainty that harmful or offensive contact or apprehension will take
place
i. he doesnt care, he just wants the chair
ii. according to the restatement, if he doesnt know itll be harmful or offensive, its not an
intentional tort
iii. Garratt v. Dailey
1. Trial court focused on whether Dailey (5 yr) intended to harm Garratt, but for an
intentional tort, court should have used the 2nd definition that Dailey intended
to pull out the chair and knew with substantial certainty that it would lead to
harmful/ offensive contact (i.e. that Garratt would sit down). The court didnt
pay attention to the other two definitions.
2. Desire to cause harm and knowledge of substantial certainty are independent of
each other
a. BOTH are grounds for imposing an intentional tort liability

c) Knowledge to a substantial certainty that MERELY CONTACT will take place and the result is
harmful or offensive to a reasonable person
i. Subjective intent, what he was thinking
ii. Twerski doesnt like this definition of knowledge to substantial certainty that contact
will take place AND that contact incidentally turns out to be offensive or harmful. Why
should we have a tort that he knew shed sit in the spot? (If Dailey knew Garratt would
sit where chair had been that is sufficient to make out intentional tort)
iii. Troubling that court says if he had a desire to harm, he would also have the knowledge
with subst certainty that harm would take place (not nec true)
1. Twerski poses the question- Why do we even bother with his intent/what was in
his mind if the kid is unaware that the contact is harmful or offensiveThis is a 5
year old kid.
2. Twerski thinks the definition should include intent to harm
iv. Case is saying it doesnt matter if he knew shed be offended.
d) Pg. 16 Hypos 1 and 2
i. Ranson v. Kitner
1. shot s dog thinking it was a wolf. Court holds liable for trespass to chattel
under third theory of intent HAD INTENT TO CAUSE A CONTACT, GOOD
FAITH DOES NOT MATTER.
Page 1 of 60

2. Even though intention was good, intent to contact resulted in a


harmful/offensive contact, so held liable
3. Had knowledge to substantial certainty that you were going to make any
CONTACT. (third definition)
4. Def acted reasonably under the circumstances but killed wolf intentionally
5. Authors dialogue pg 16
6. Contributory neg isnt a defense to an intentional tort
7. TW: Why are we imposing liability based on intent when his intent was to shoot
a wolf?

Transferred Intent:
a) imposes liability when a defendant has the intent to commit any of the 5 intentional torts
(assault, battery, false imprisonment, trespass to land, or trespass to chattels) and harm results
to anothers person or property
i. Intent carries over from one tort to another tort; and one person to another person
ii. Talmage v. Smith
1. Transferred intent because he meant to throw the stick at one boy on his roof,
and hit another instead, which ultimately blinded him
a. Wouldnt be held liable if he didnt mean to hit the boys, only meant to
scare them (throw it at feet or nearby tree) because they were
trespassers OR if he used reasonable force
b. Used unreasonable force AND he intended to hit one of them, so he is
liable (then why liable in Ranson?)
i. Maybe because in Talmage thers trespassing, not in Ranson
c. Theory: if you are an intentional tortfeasor, dont come crying when you
hurt the wrong person
iii. Palsgraf
1. liability of neg to an unforeseen plaintiff (Talmage)
2. helped establish the concept of prox cause, a limitation of neg
3. guard trying to help drops a box that he didnt know contained fireworks and
they exploded and injured a woman
4. the relationship of the guards actions was too indirect for liability
iv. Hypos pg 21**
b. Intent in other contexts
i. Concept of intent used in various torts
ii. Many liability insurance policies exclude coverage for intentional torts
iii. TW: what kind of intentional torts did them mean to exclude?
iv. Would a liability insurance policy cover a 5-year-old pulling a chair?
1. Not what the exclusion was meant to cover
v. Mandated Workers compensation
1. If the person is getting workers compensation, the company is immune for tort
liability UNLESS the employer intentionally harmed them
2. Most injuries that take place, you cant sue
3. In the concept of workers comp, what does intent mean?
4. Helf v Chevron
a. a plaintiff may not demonstrate intent by showing merely that some
injury was subst certain to occur at some timethe employer must
Page 2 of 60

know that the assigned task will injure the specific person that
undertakes it

B.)

Battery
Definitions Restatement 13 and 18
i. An actor is liable for battery if he intends to cause a harmful and offensive touching with
the other or third person, or an imminent apprehension of such contact AND harmful or
offensive contact directly or indirectly results
1. negligence or recklessness is not enough- must be intentional
ii. the contact is not consented to by the other or consent is procured by fraud or duress,
and
iii. the contact is not otherwise privileged
iv. Based on Reasonable person standard. Super-sensitive P does not count. If P really
hates to be touched, he cant sue for battery if someone bumps into him
v. You can find out about battery after (sleeping beauty)
a) Brzoska v. Olson
i. s claim they suffered mental distress from offensive contact from dentist who had
AIDS
1. this was not a contact that would be offensive to a reasonable person, b/c there
wasnt any actual exposure to AIDS
a. reasonableness is a question of fact for the jury
2. RULE: Need some intent from (remember 3rd definition only is intent to
contact); and must experience a harmful or offensive contact, which is
offensive to a reasonable person ( cant be overly sensitive). Court here uses
third definition.
vi. window of anxiety btw call about AIDS and result of test
ii. Twerski: thinks this should be a battery (could have been reasonably offensive the
Department of Health called to tell people to get tested. We dont need to see the
contact as offensive at the moment of contact (i.e. kissing a sleeping girl). Court just
didnt want to feed AIDS hysteria, so its decision was based on policy.
iii. HYPO 5 and 6 pg 30**

Scope:
a) Eggshell Skull Rule
i. If one batters a plaintiff with an eggshell skull (or a peculiar vulnerability to an injury),
P may recover all damages notwithstanding the lack of foreseeability of the ultimate
injury. Once battery is established, is liable for all ensuing damages
b) Battery is extended when a defendant makes contact with something in plaintiffs physical
possession
i. Fisher v. Carrousel Motor
1. Fisher desired to cause offensive contact by touching the plate
2. Although there was no contact with Fishers person, knocked a plate out of his
hand (seen as an extension of person) and the contact was offensivetherefore
it was enough to make out a battery.
3. First case we have with punitive damages
4. Respondeat Superior: was acting in a managerial position, thereby making the
corporation liable (as long as within the scope of employment)

Page 3 of 60

C.)

D.)

Assault
Definition (Restatements 21):
a) Intent to cause harmful or offensive contact (battery) or an imminent apprehension of such
contact; AND
b) Person is put in imminent apprehension
i. Involves a who touches the mind of the . Actual ability to carry out intended harm
is not necessary
Scope:
a) Apprehension alone does not make out an assault; must have necessary intent
b) must be AWARE of the danger (kissing a sleeping girl doesnt count for assault)
c) Words alone do not constitute an assault
d) Battery verse Assault
a. Every battery includes an assault (TW SAYS NOT TRUE), but not every assault includes a
battery
e) To constitute an assault, its unlawful attempt to commit a battery, incomplete because of some
intervening cause

Conditional Threats:
a) Assault needs to put in imminent apprehension of bodily contact. It is satisfied when makes
a conditional threat (give me money or Ill kill you).
b) Usually words alone will not be sufficient; must make some kind of physical gesture.
c) Threats about the future wont work because apprehension isnt immediate

Western Union Telegraph v. Hill --
a) worker wanted to love and pet woman. Employee reached for , and scared her because she
thought he was going to sexually assault her
b) had to make out 1) intent to commit unlawful contact and 2) creation of fear of that contact
in order to recover
i. Court finds for b/c the worker couldnt actually have reached since the counter he
was sitting behind was too big/high to get around
ii. Court just says its assault if he could have touched her
c) Twerski: Court missed the boat looked at whether physical contact was possible instead of at
whether the intended to cause a harmful or offensive contact at the s apprehension
i. Actor could have just argued he had no malicious intent just a misunderstanding (need
intent for assault)
b. Respondeat Superior case (not within scope of employment here, TW disagrees)

Assault need to cause apprehension of immediate harm
a) pushing a door open without knowing theres a person on the other side would not be assault
because theres no intent

False Imprisonment
Definition:
a) an actor is liable for the tort of false imprisonment when he intends to confine another within
fixed boundaries
i. and his act directly or indirectly causes such confinement
ii. and the other person is conscious of the confinement OR is harmed by it (cant imprison
or assault someone in their sleep)
Page 4 of 60

Elements:
a) Willful Detention
b) Without Consent
c) Without Authority of law
i. Shopkeepers Privilege allows a store owner to hold a customer
1. Must have reasonable belief he is stealing (Hold him for reasonable time and in
a reasonable manner)
2. Shop Keepers Privilege is only available if the person actually stole something, if
he didnt then, no privilege and can sue for FI
a. Shopkeepers are taking a gamble that they are right

Grant v. Stop n Go Market of Texas
a) man accused of stealing cigarettes/shoplifting at gas station. Store argued that they did not
willfully detain, and that man was free to leave. claimed that he was forced to stay. Store
claimed that it had the Shopkeepers Privilege
i. had to know with substantial certainty that he was confining in order to there to be
FI
ii. Did Calhoun intend to restrain him? Did he tell him he couldnt go? Jury question
iii. Its a jury question whether or not Grant consented to stay (coercion might be a factor)
iv. Twerskis Thoughts: Unless the shopkeeper communicates that is free to leave, the
situation is highly coercive and is probably false imprisonment. Now a matter of fact for
the jury to decide (though he thinks this is unfair)
b. Authors dialogue pg 42**
c. McKinneys General business law
i. Defense for shopkeeper if they are detained for a reasonable amount of timethe time
necessary to ask them if they stole
d. HYPO pg 45**



E.) Intentional Infliction of Emotional Distress (IIED)
Definitions:
a) Conduct must be intentional or reckless
b) Conduct must be extreme and outrageous
c) Causal connection between wrongful conduct and ED
d) Emotional Distress has to be severe

Scope:
a) The burden is very high on to prove that the distress was severe and the conduct was extreme
and outrageous
b) Usually the jury decides if its severe/outrageous but with this tort, many courts will direct
verdicts in these cases because it affects free speech
c) Can have transferred intent---answers phone and hears call of son dying

Harris v. Jones
a) Supervisor taunts Harris for his stutter. Enough to make out case for IIED?
i. Court holds that his ED wasnt severe, so they do not look at the other elements
Page 5 of 60

ii. Easier for courts to get rid of cases on the narrow reading (easy to say not severe of
extreme/ outrageous then prove the other elements of IIED).
iii. Insults dont count as outrageous misconduct
iv. Authors dialogue pg 52**

First Amendment and IIED
a) Court will throw out IIED cases if they infringe on 1st amendment rights of free speech
i. Synder v. Phelps (2011) WBC protesters present at Synders funeral (soldier killed in
Iraq). Father sued for IIED, although he made out all elements of the tort, ultimately
court said he couldnt recover b/c the speech was of public concern
1. not a private grudge against the soldier
ii. Can have IIED, but if it violates a constitutional right, will not recover
iii. How far can you push these decisions?


F.) Trespass to Land
Definition:
a) Intentionally entering onto anothers or forcing another to enter onto someones land
i. Do not need to know it was someone elses land
ii. Simply must trespass (act) intentionally.
iii. NOT if P didnt mean to enter the land (lost control of their car)
iv. failure to remove a thing placed on the land pursuant to a license or other privilege after
license is over is also trespass to land
Scope:
a) Wrongfully remaining on land, even if you entered lawfully, also constitutes a trespass
i. Rogers v. Kent County
1. Man ran over a snow post for a fence that was left in his land with his lawn
mower and was killed. found liable for not removing stake even though it was
allowed to have originally placed it on land.
2. Restatement says you can have trespass by continued presence once contract
has been terminated
ii. Twerskis problem with this case: How is it an intentional tort? Simply missed removing
one spike
1. TW: is this an intentional tort or negligence?

G.) Trespass to Chattels and Conversion
Definition:
a) Difference between Trespass to Chattels and Conversion depends on the seriousness of the
offense
b) Trespass to Chattels:
i. Intentional interference of anothers personal property but no serious repercussions
ii. Minor interference- only pays the value of the harm caused to the chattel

c) Conversion

Page 6 of 60

i. Intentionally take chattel that you know isnt yours, or mistakenly take anothers
property for a prolonged period of time. The issue is SERIOUS INTERFERENCE WITH THE
OWNERS ABILITY AND RIGHT TO USE IT
ii. Required to pay full market value for the chattel, or if the owner wants the original back,
then returns chattel + cost of damages
iii. Important factors in determining seriousness: 1) extent and duration of interference, 2)
intent of actor, 3) duration or resulting interference w/ others right of control, 4) harm
done, 5) inconvenience/expense to the original owner
d) Both Trespass to Chattels and Conversion REQUIRE DAMAGE
e) Pg 58 HYPOS***

f) Intel Corp v. Hamidi
a) disgruntled former employee sends Intel workers emails about the company
i. No harm to or disruption of the system. Intels main concern was that the emails
kept being sent (not property damage)
ii. one who intentionally intermeddles with anothers chattel is subject to liability
only if his intermeddling is harmful to the possessors materially valuable interest in
the physical condition, quality or value of the chattel, or if the possessor is deprived
of the use of the chattel for a substantial time, or some other legally protected
interest of the possessor is affected (p. 49) NEED HARM
iii. Twerski says: really a public policy issue; shouldnt have been decided with trespass
to chattel because it requires damages to real property. Might need a special rule
for internet related issues dealing with invasion of property. Why does the court
require harm? That isnt a current inquiry with internet issue today because there
wont be physical harm.
b. Authors dialogue pg 64**
c. HYPO pg 66**

H.)

Umbrella Intentional Tort


a) proposed 5 to Restatement (Third) of Torts:
i. an actor who intentionally causes physical harm is liable for that harm
ii. attempt to expand liability so that intentional harm of all types are covered
iii. Twerski thinks this is a bad idea, could risk bringing in far reaching suits. Should keep to
the doctrine of defined elements
iv. HYPOS pg 67**


Chapter 2: Privileges/ Defenses

A.) Privileges/Defenses generally


a) The elements in intentional torts make up the plaintiffs prima facie case
b) Privileges make normally prohibited conduct allowed under certain circumstances
c) functions as an affirmative defense
d) bears burden of proof
e) most deflect tort, consent negates the tort

B.) Consent
Definition (Restatement 892- Meaning of Consent)
Page 7 of 60

a) Def can also prove the plaintiff had a willing state of mind even if no consent actually
communicated
b) Willingness in fact for conduct to occur.
i. May be manifested by action or inaction and need not be communicated to actor
ii. If words/ conduct are reasonably understood by another as consent, then they
constitute apparent consent and are as effective as consent in fact
iii. May be express or implied
iv. Invalidated by fraud, duress, or lack of capacity to consent (ex: if adult obviously suffers
from temp/perm mental disability or if minors consent to potentially harmful conduct)


Scope:
a) Implied Consent
i. OBrien v. Cunard Steamship Co.
1. Implied consent from her behavior: was waiting in line with the others, raised
her arm, was on the ship and needed to get off and would have been
quarantined without the vaccine.
2. Inaction can be consent (she didnt tell the doc she didnt want the vaccine).
Doctor reasonably believed she consented in this situation
3. Twerski: this is an informed consent case; doctor took away her choice. Its
different from Ranson (shooting dog that he thought was a wolf), b/c the doctor
had clear opportunity to communicate and get consent. But how much
communication is enough? It has to be reasonable Twerski thinks there should
be a higher level of communication.
4. He could be guided only by her overt actTW says not true, he could have
asked her
5. how clear does communication have to be? Situational
6. HYPO pg 77-78**

b) Assumption of the Risk
i. Hackbart v. Cincinnati Bengals
1. NFL Player intentionally struck in the back of head by opposing player after the
whistle blew (clearly outside the rules of the game). Court has to decide if this
falls into the activity that pro athletes should expect when they assume the risks
of the game.
a. Rule: Its highly questionable whether a professional football player
consents or submits to injuries caused by conduct not within the rules,
and there is no evidence that the court has seen which shows this.
2. Twerski: this case shouldnt be in tort, should be a question of contract or
resolved within the NFL, they know their rules best. Pro athletes are getting paid
good money and they know what theyre getting themselves into
3. How broad is the consent that an athlete gives to violence in the game?
4. HYPO pg 84

c) Informed Consent
i. Christman v. Davis
2. Doctor does less than what was consented to
Page 8 of 60

a. Rule - If the doctor chooses to perform a less invasive procedure than


the one discussed, its not battery or medical malpractice
b. Medical battery exists where a doctor obtains consent of the patient to
perform one type of treatment and subsequently performs a
substantially different treatment for which consent was not obtained
(like operating on left ear when doc had only gotten consent for
operation on the right See: Mohr v. Williams)
c. Patient wouldnt have said yes if she knew it was less invasive
procedure
3. Your medical judgment is important but the patients autonomy to agree to
something is more important
4. Twerksi: Disagrees with the court, thinks this should be battery. The patient
asked for the full procedure; half procedure wasnt consented to
5. Authors dialogue pg 87***

d) Consent is invalidated if obtained by fraud
i. DeMay v. Roberts
1. Doctor has someone come with him to a patients house during her home
childbirth. The family assumes hes a medical assistant and the husband lets him
in. Consented to his presence under false pretenses/ fraud
2. In this case, the def has to communicate/educate the plaintiff
3. Informed consent-what does the def have to tell the plaintiff so the plaintiff can
make an informed decision?
4. According to Restatement 892(B)(2) the misrepresentation must be about
something that affects the intrinsic nature and quality of the invasion or the
harm, cannot be about some collateral fact
5. Twerski: doesnt think the cause for battery is very strong, could have been
privileged b/c woman was about to fall. How much does the doctor really need
to say? The wife/husband could have asked who he DeMay was instead of
assumed. Could be a trespass to land
6. HYPO pg 93**

f) Consent is invalidated if obtained by duress
i. Grager v. Schudar
1. While in prison, woman was sexually assaulted by one of the workers. Issue of
whether Grager consented to the act or notlegally she cant consent. Also,
consent is invalidated if obtained by duress (jailer was in a position of authority,
could she really say no?)
2. Let jury decide

g) Informed Consent (contd)
i. Scott v. Bradford
1. consented to hysterectomy surgery from but suffered severe complications,
which did not warn about beforehand. filed suit against , claiming failed
to advise her of the risks involved or available alternatives to surgery. Had she
been properly informed, she would have refused surgery.
2. Not case of medical malpractice
Page 9 of 60

ii. Court says not battery because he did what she consented tocould be negligence for
failure to inform the patient of options, regardless of due care exercised at treatment
(doctor wasnt neg, he just didnt inform her of risks)
iii. Questions to ask yourself as plaintiff
a. Was there adequate info given to the plaintiff?
b. What legal test are we using to determine how much information is necessary?
iv. Creates duty of informed consent : doctor has duty to inform/educate patient of the
alternative options and the risks for medical procedures
1. Considers Reasonable patient (est. in Canterbury) vs. Reasonable doctor
standard (different states adopt diff standards in their informed consent
statutes)
a. MOST adopt reasonable doctor standard (hes the one with the
knowledge, its less likely the patient will know what kinds of questions
to ask)
v. Informed consent is about autonomy
1. cannot deprive patient the right to make a free, informed choice
2. Elements necessary for informed consent:
a. Duty to inform patient of material risk
b. Causation (MUST PROVE DECISION CAUSED INJURY)
i. Two causation standards: reasonable patient, actual
a. Reasonable patient: must show a reasonable
patient would have made a different choice. This
essentially takes away the s autonomy
b. Actual : Saying the actual patient would have
made a different choice if he had been informed.
has to convince jury of this.
c. Injury
i. Standards the courts apply for materiality
a. Reasonable doctor test: Whether a reasonable
doctor would have felt it necessary to inform the
patient (Based on the custom of profession)
b. Reasonable patient test: What a reasonable patient
would want to know prior to making choice
vi. How much info is needed? Determined by patient. Has to be enough to make an
intelligent decision. Exceptions:
1. Risks that are known to everyone
2. If detrimental to patients well-being therapeutic privilege(however,
undercuts informed consent, so hardly ever used)
3. Emergency situation and patient cant decide whats best
vii. Twerski proposes: broadening of informed consent to recognize a duty to provide
statistical information about treatments and the risks.
viii. Trial was right about amount of info test (reasonable patient) but wrong about
causation test (patient) TW says this is a reasonable decision
ix. In medical malpractice, a patient suing under informed consent must prove
a. Doctor failed to inform
b. He he had been informed, he wouldnt have done it
c. The adverse consequences not known did happen and he was injured because of it
Page 10 of 60

d. HYPOS pg 105**
e. Authors dialogue 6 pg 108
i. TWTroubled in Scott v Bradford by its summary of the elements that a plaintiff
must establish to make out an informed consent case
1. Def failed to inform the plaintiff about a material risk
2. The plaintiff, if informed, would not have consented
3. Adverse consequences that were not made known actually happened and
caused the injuryhow do we assume the surgery caused the injury? What
would have happened if she didnt do the surgery? She could have died
otherwise so its hard for tort law, which provides for damages that
compare the plaintiff in the uninjured state to the injured state
ii. JYoure wrong. Once the doctor decides to go it alone and not consult the patient,
and the very harm happened, causation is established.

C.) Self Defense
Definition (Restatement 63 and 65)
a) Restatement 63: Self-Defense by Non-Deadly Force
i. Privileged to use reasonable force, not intended or likely to cause death/ serious bodily
harm to defend against offensive contact that he reasonably believes is about to be
inflicted on him.
b) Restatement 65: Self-Defense by Deadly Force
i. Can defend against force likely to cause death/serious bodily harm when he reasonably
believes that
i. Other person is about to inflict intentional contact or bodily harm
ii. Hes put in peril of death or serious bodily harm
ii. This defense exists although the actor correctly or reasonably believes that he can safely
avoid the necessity of so defending himself by
i. Retreating if he is attacked within his dwelling place, which is not also the
dwelling place of another
ii. Permitting the other to intrude upon his dwelling
iii. Abandoning an attempt to effect an lawful arrest
iii. The privilege does not exist if the actor correctly and reasonably believes he can with
complete safety avoid the necessity of defending himself by
i. Retreating if attacked in any place other than the place of his dwelling
ii. Relinquishing the exercise of any right or privilege other than his priv to prevent
intrusion of his dwelling or effect lawful arrest
c) Courvoisier v. Raymond
i. Raymond reasonably believed he was being attacked and burglarized; he shot a police
officer (thinking it was a rioter) in self-defense. Case of a mistake.
ii. To be within the privilege of self-defense you must show: (to jury)
i. You made a good faith effort to protect yourself
ii. Fears were reasonable under the prevailing circumstances
iii. The force was used reasonable.
1. Failure to act reasonably is a BATTERY, not negligence
iii. Why should an innocent person have to suffer?
Page 11 of 60

d) Authors Dialogue 7 if belief that another is about to inflict harm or offensive contact is
unreasonable, then the defendant is liable to the plaintiff for battery, but it is not liable for
negligence because self-defense is an affirmative defense with the burden of proof on
defendant while the burden of proof for negligence is on the plaintiff

e) Compare to Ranson (where thought he was shooting a wolf, but was held liable), here
Courvoisier is REACTING, not ACTING. He thought he had to shoot to save his life; Ranson didnt
have to shoot the wolf. This is more like Talmage where was permitted to respond reasonably
to trespassers

f) Restatement 70 and 71 limit the privilege of self-defense to the amount of force the actor
reasonably believes is necessary to protect himself, and will be liable for excessive force.

g) When self-defense is no longer a privilege-
i. loses right to self-defense if he starts the fight, unless he abandons the fight and gives
notice to the other person that hes doing so.
h) Hypos 20 and 21 pg 114

D.) Defense of Others
a) Privilege of defense is extended to third parties to the extent that the person is being threatened
(same rules as self-defense: cant use excessive force, etc.)
b) Privilege is most often invoked when acts to protect family members/ friends against third
persons. (Some courts say it applies to strangers)
c) What if you made a mistake and the person didnt need help?
i. Some courts allow the privilege if the mistaken belief was reasonable. Others refused to
extend the defense of others privilege when its a mistake.

E.) Defense of Property
Definition:
a) You can defend your property by reasonable means
i. Law does not allow deadly force to defend property
ii. BUT, If you are seriously endangered while trying to defend you property than you can
use deadly force (Becomes a self-defense privilege)
iii. Katko v. Briney
a. landowners set spring gun up in a house where no one lives to prevent
trespassing. The gun shoots someones leg off as hes trying to steal. s found
liable.
b. Risk-benefit analysis
c. Rule: life is more valuable than property
i. Though Posner says this shouldnt be an absolute rule, it should be
situational. We should use a standard of reasonableness that permits
the courts to weigh such considerations as the value of property at
stake. (Twerski tends to agree; thinks the maxim is too simplistic. There
are lots of times where we say property is more valuable than life, think
products liability a company wont spend $10 for safety to save $9 of
injury)
b) Recovery of Property
Page 12 of 60

i.

Privilege to use reasonable force to regain a chattel tortuously taken by another so long
as the rightful possessor acted promptly in hot pursuit after the dispossession or after
timely discovery of it.
a. Dont have privilege if theres no immediacy
b. Using force of deadly/serious bodily harm is not permitted
c. Have to be right about possession being taken
1. If wrong, it could be a battery.
d. Hey! Thats my purse!

F.) Necessity
Definition:
a) Privileged to utilize the property of others to prevent damage to property under circumstances of
necessity, but must pay the damages b/c you made a decision that your property was more valuable
that anothers.
i. Not a tortfeasor in this situation, but still economically responsible.

b) Vincent v. Lake Erie --
i. Necessity of ship to be moored: would have been destroyed in the storm had they tried
to move it when they finished unloading their cargo
ii. Captain acted reasonably under the circumstances in actively keeping the boat moored
but is being held liable because he was in a better position to know the value of the ship
v. value of dock. The dock owner was innocent party. Ship owner acted intentionally by
doing cost/benefit analysis and is therefore liable for damages.
a. Twerski: says this is a policy consideration. Have to allow some form of recovery
or dock owners will frustrate rescue.

c) Privileged to damage property in order to save lives or communities (necessity of the situation
justifies purposeful damaging of property)
i. Surocco permitted to damage s home to prevent massive inferno engulfing a city.
could not recover (Why is this? How is this different from Vincent?)
d) Hypo 22 pg 127
e) Legal Authority
i. Considers privileges retained by public officials.
ii. When such officials act within the limits of their predefined roles, they are not liable.
f) Disciplining Children
i. Law allows people to use reasonable force in disciplining children.
ii. HYPO 23 pg 129

G.) An Umbrella Justification Defense
Definition:
a) Even if conduct doesnt fit into the conventional defenses, can still work as a privilege if you are
acting reasonably based on the situation.
i. Problem is its far-reaching; can say all intentional torts are justified just b/c acting
reasonably
b) Sindell v. New York City Trans
i. Bus driver has a group of rowdy kids on his bus; he cant get them to stop, so he drives
them to the police station. Takes the kids to a place not consented to. Is he justified
and not liable?
Page 13 of 60

1. SC allows this defense. Reasonable force allowed to defend property and bus
driver has in loco parentis rights (acting as care-giver/ custodian)
2. Twerski: thinks umbrella defense is dangerous b/c could absolve virtually all
liability. Why not just treat this as a tort; could be false imprisonment

Chapter 3: Negligence
A.) To have a cause of action for negligence, must prove 5 Elements:
a) Duty (in general) people have duty to act reasonably to avoid causing harm to another
i. However, no duty to rescue a stranger
ii. Always decided as a matter of law by the court.
b) Breach of Duty Once there is a duty in place, must show failed to act reasonably under the
circumstances.
i. Jury must decide what constitutes reasonable care under the circumstances then find
that the failed to meet that standard.
ii. Most commonly decided by juries between 2-8 range. If its above 8 it may be a directed
verdict.
iii. Consider B<P L (if the burden is less than the probability of the harm times the severity
of the injury, then there is no breach)
iv. B- Burden of preventing harm; P- probability that the harm will occur; L- gravity of the
harm
c) Cause-in-Fact must show a connection between s negligent conduct and the harm
suffered.
i. But for the s conduct would the harm have occurred?
d) Proximate Cause the conduct has to be closely related to the risk taken to determine it caused
the harm.
i. Ps injury must be foreseeable, within the scope of the risk
e) Harm must suffer actual, tangible harm to make out a prima facie case for negligence

B.) Risk Utility Balancing
a) Risk/utility consideration isnt given to the jury as part of instructions. Juries only decide what is
reasonable. Judges do the balancing and consider the standard of care on a 10 point scale
anything under a 2, judge can step in and say we must hold the liable / above an 8, judge steps
in and says not liable anything between a 2 and an 8 is sent to a jury.
i. Mostly looking at policy and economic assessment- if the burden would be too great to
put on the , judge will step in and direct the verdict.
ii. Negligence cases are very fact sensitive. Usually sent to jury and have them make a
commonsense assessment on whether s conduct was reasonable. Courts do overturn
jury decisions saying reasonable persons cant differ, but its rare.
iii. Jury is making law for a particular case (Twerski: same case facts could be in two
different court rooms and there could be opposing verdicts).
iv. Twerksi: shouldnt treat B < P L as dispositive (harsh to find life/limb could be sacrificed
for economic efficiency)
v. If the person has inadequate data about the risk, the question becomes whether a
reasonable person should have invested more resources to learn about potential risks
before acting.
vi. In general, risk-utility balancing is for the jury
Page 14 of 60

vii. Authors Dialogue 8 pg 146:


1. TWThe B<PL test about economic efficiency is better than being safer is weird
2. Jlearned hand himself knew that his formula was just an aid in thinking about
reasonable conduct.
3. TWrisk utility assessment is getting better every day
a. If the court would have just asked if the acted reasonable, the
would have lost.
viii. HYPO 24 pg 150
ix. Authors Dialogue 9 pg 152
1. TWSo much attention is paid to this balancing but the jury is kept in the dark.
2. JThe risk utility test is for the judge so that she can decide whether to direct a
verdict. We dont need juries trying to do economic balancing.
3. TWThe law is the s friend do s want juries to know more about the law
(opposite for )

b) Risk Utility Balancing in Restatement:
i. Restatement, 2nd 291 Unreasonableness; How Determined; Magnitude of Risk and
Utility of Conduct:
1. The risk is unreasonable and the act is negligent when the risk is of such
magnitude as to outweigh what the law regards as the utility of the act or of the
particular way in which it is done.
ii. Restatement, 3rd 3 Negligence:
1. A person acts negligently if he doesnt use reasonable care under all the
circumstances. Factors to consider for reasonable care: 1) foreseeability of the
harm; 2) foreseeable severity of the harm; 3) the burden of taking precautions
to eliminate/ reduce risk of harm.

c) Lubitz v. Wells dad leaves golf club out in the back yard, his son hits friend in the face with it.
Does the father have a duty to put things away?
i. Court says they are not going to impose a duty, so they cut the case off.
ii. Twerski: Saying no-duty is like using a HATCHET. To send to the jury and see if the
duty was breached is taking out the violins and telling all the facts of the case.
iii. Policy reason: not going to impose standard of care to put away common objects on a
persons own property. Says a golf club is not obviously and intrinsically dangerous
iv. Parents usually not liable for the torts of their children.

d) United States v. Carroll Towing bargee (employee) is absent from boat, it floats away from
dock, springs a leak and sinks. Results in damaged cargo and boat damage.
i. Learned Hand introduces B < P L in this case.
ii. Question of whether the bargee was negligent in leaving and therefore liable for
damages
iii. Although comparative fault didnt really exist in all areas of law until the 1950s, it always
existed in admiralty law could reduce damages by arguing contributory negligence.
iv. Decide since the bargee didnt have a legitimate excuse for being gone for as long as he
was, he acting unreasonably; therefore negligent

Page 15 of 60

e) Washington v. Louisiana Power & Light Co. radio hobbyist gets electrocuted and dies from an
exposed wire in his backyard. He had been shocked 5 years earlier and knew of the risks/ was
very cautious around the wire.
i. Jury finds for . On appeal, court steps in at standard of care stage and says not liable
b/c burden of burying all the wire in town is far greater than risk of this getting
electrocuted. Saying reasonable persons could not differ that the conduct of the electric
co. was not negligent.
ii. Though High L, P is low and High B = no liability
iii. Two things going on:
1. Putting data into the formula
2. The weighing process (objective)
iv. Two questions:
1. Did the act reasonably given your assessment of the risk? (if no, this is the
only question)
2. Should you have taken steps to get additional information? Were you negligent
in not getting more information before acting?
a. Should they know or should they have known? when asking this,
youre doing risk utility balancing (burden of getting information)


C.) Qualities of Reasonable Persons
a) Knowledge- Liability for Foreseeable risks
i. When the actor might have inadequate data about the probability or gravity of a bad
result then have to ask if a reasonable person should have invested more resources to
learn about potential risks before acting.
ii. Vaughn v. Menlove guy tries to prevent his hayrick from burning and damaging
neighbors prop by building a chimney. Makes matters worse. Good faith effort doesnt
get you out of negligence if you should have known.
iii. Delair v. McAdoo is driving his car and the tires are worn way down. Gets into an
accident with . Court finds for , saying was negligent in not keeping his car in a safe
condition, which is a duty he owes to others on the road. Court concludes must
know condition of his car
1. Twerski: thinks must know language is too strong. If he had taken reasonable
steps to protect, then he wouldnt be negligent.
iv. HYPO 25 pg 155

b) Knowledge of Professionals
i. Restatement, 3rd, 12 Knowledge and Skills:
1. If an actor has skills or knowledge that exceeds most others, then they are
circumstances to take into account in deciding if actor behaved as a reasonably
careful person.
2. (as far as people with below average knowledge, they can still be liable because
they should try to get knowledge)
c) HYPO 27 pg 157
d) How the Reasonable person responds to Emergencies
i. If actions are prompted by an emergency, then that can serve as an adequate defense
to negligence
Page 16 of 60

ii. Cordas v. Peerless cab driver jumped out of the car to save himself and car hits
people. Court finds he acted reasonably given the emergency situation.
1. In emergency situations you dont have time to get info (P). Cost of staying in
car to gather more data was very high relative to the risk of being shot by the
mugger
2. Twerski has a problem with this:
a. No need for an extra doctrine b/c the emergency will be built into the
B< P L analysis
3. Compare to Vincent v Lake Erie
a. He had a right to be there but had to pay, why didnt cab driver have to
pay? Difference between intentional torts and negligence. In Vincent,
he was an intentional trespasser. The difference is the intent.
iii. HYPO 28 pg 161

e) Does the reasonable person follow customary practices?
i. Custom gives answers to the B < P L questions on standard of care. Custom creates
strong presumption. Its relevant but not always binding (still have to show the custom
is reasonable)
ii. If you depart from an industry custom you have little chance with a jury, only real hope
is to show custom doesnt apply to case at bar (Twerksi: if doesnt follow industry-
wide custom hes a shmuck and case is dead in the water)
iii. Trimarco v. Klein shower door shatters and injures . Custom changed to use shatter
proof doors if putting new in or fixing- but NOT TO REPLACE all doors.
1. You dont have to use B < PL because custom is standard of care.
iv. Twerski issue with custom evidence:
1. Custom evidence is damning and stigmatizes the defendant
2. Must be proper situation to introduce it because it, in effect, shifts the burden
from plaintiff to defendant
v. HYPO 28 pg 161
vi. Authors Dialogue 10 pg 162
1. Jim: you believe that the fact that an actors conduct took place under an
emergency is merely one fact in risk-utility balancing to determine if the actor
was reasonable
a. Even in emergencies, the actor can be negligent if he made a bad choice
b. It seems to me the court is saying that where his life is threatened, the
cabbie isnt required to be a hero and take a chance of sacrificing his life
in order to prevent risk of harm to othersthis has nothing to do with
emergency
TW: Im not sure, if the court is saying that one is permitted to place anothers
life in jeopardy in order to save his own, I dont see the diff btw this and Vincent
v Lake Eriein that case it was a priv to take anothers property but they still
had to paywhy doesnt the cabbie have to pay?

Jim: all human activity can put people at riskif the necessity priv is taken
beyond intentional torts, itll destroy the idea that there ought to be no liability
for non-negligent conduct

vii. HYPO 29 pg 166
Page 17 of 60

viii. Authors Dialogue 11 pg 167


1. Jimthe custom of the industry to replace regular glass with shatterproof glass
for shower doors was clearly relevant to whether the should have replaced
the shower door in the s apartment. The custom shows that the harm was
foreseeable, that the replacement was feasible and no expensive.
2. TWIf, in Trimarco, there was evidence that building owners were replacing
regular shower doors with the shatterproof doors as a matter of course, then
custom evidence should have come in, but that wasnt the case.

f) Physical/ Mental Attributes of Reasonable Person
i. Handicap --
1. Roberts v. State of Louisiana Blind man is not using his cane, which is a
custom in familiar places. Bumps into old guy who gets injured. Were not going
to hold a handicapped person to a higher standard of care. Still has to act
reasonably under the circumstances.
a. Still doing B < P L; probability of the harm is low b/c this man has walked
to the bathroom successfully w/o a cane many times.
b. Weighing process
i. Objective standard of reasonableness after the facts are
considered
1. Only objective standard, everything else is subjective
(two exceptions: mental incapacity/kids)
2. Do we let them off or use reasonable person standard?
We dont take mental incapacity into account, we just
use reasonable person standard
3. Jury will take mental incapacity into account and wont
reduce plaintiffs fault by a lot
2. Authors Dialogue 12 pg 177
a. TWwhy doesnt the same argument apply to people with mental
illness?
b. Jwe cant lock up children until adulthood
i. If the mentally ill cant conform to normal standards, their
caretakers should keep them out of harms way
c. TWyoure wrong
i. Today, involuntary commitment is limited ot cases where the
person is dangerous
ii. The decision now is to put the mentally ill into society
iii. If kids are part of our world and dont have to measure up to
adult standards, why do the mentally ill?
d. Jif society decided to deinstitutionalize the mentally ill and integrate
them into our world, then they should be treated as all other members
of society
i. Maybe its because kids arent given FULL entry into adult world
ii. Children
1. Stevens v. Veenstra 14 y/o is driving a car for drivers ed. He hits . Courts
hold children to a lower standard of care (they have less info/ they dont know
how to assess harm and cant say they should have known b/c that only
comes with age)
Page 18 of 60

a. However, if they are performing adult activities (like driving), they will
be held to the same standard.
2. Restatement, 3rd 10: A childs conduct is negligent if it doesnt conform to
reasonably careful person of same age, intelligence, and experience. BUT
a. A child less than 5 is incapable of negligence
b. Doesnt apply when child is engaging in dangerous activity that is
typically undertaken by adults
3. Hybrid test for children
a. Age, intelligence and experience to see if they acted reasonably
4. HYPO 30 pg 176

g) Standard of Care for Professionals
i. Having held yourself out as a professional, you have the obligation to acquire the
industrys standard level of knowledge. In other words, you have the obligation get
good data.
ii. When evaluating a Ds negligence, we can no longer ask what they should have known.
All professionals should have known and parties will use experts to testify to that
effect
iii. Cant determine the professional standard of care without expert.
iv. The judge will not give a simple reasonable person instruction but instead asks the
jury whether the def acted with the skill and knowledge normal to the profession.
v. Esp in regard to med mal, custom is used as binding
vi. Boyce v Brown
1. She sues doctor for not taking x-ray.
2. Have to prove if you would have taken X-ray, you would have prevented the
harm.
a. Did he violate the professional standard?
b. Did the violation cause the harm?
vii. Helling v. Carey Eye doctors fail to diagnose young woman with glaucoma for years.
The probability was very low, but the test was not burdensome at all. However, wasnt
the medical custom to test young people. Court directs the verdict and finds doctors
were negligent (goes against the med expert testimony presented)
1. Twerski thinks this is a slippery slope if the rule is not specific to this case- do
doctors then have obligation to use all inexpensive tests to rule out every
ailment? Seems that the burden is too high.
2. This IS a case where the court set the standard of care.
viii. HYPO 31 pg 184

D.) Judicially Determined Standards of Care
a) Outliers Between 0-2 and 8-10; when the court decides reasonable persons cant differ, judge
steps in and directs a verdict. They are taking away standard of care questions away from the
jury.
i. Making policy decisions dont want to give the case to the jury to decide
b) Timpte Industries v. Gish worker falls into a trailer and gets hurt. Says the reason he fell was
the trailer was negligently designed. SC determines trailer wasnt negligently designed, the
burden to make alternative models is too high, and the risks were open and obvious. Therefore
directs verdict for . Court clearly using B<PL.

Page 19 of 60


E.) Negligence Per Se
DEFINITION:
a) WAKE FOREST LAW REVIEW ARTICLE
b) Courts use safety statutes to set specific standards of care
c) Restatement, 3rd 14: Statutory Violations as Negligence Per Se:
i. An actor is negligent if, without excuse, the actor violates a statute that is designed to
protect against the type of accident the actor's conduct causes and if the accident victim
is within the class of persons the statute is designed to protect. (STATUTE ITSELF MAY
NOT PROVIDE CIVIL LIABILITY)
d) When legislature has addressed a certain issue and said conduct is inappropriate, it is stronger
than the finding of a selected jury (legislation is seen as the representative of the community)

SCOPE:
a) Statute sets MINIMUM standard of care (floor not ceiling)
b) Martin v. Herzog - s car collided with s. was driving in the middle of the road but did
not have his lights on his buggy. Statute says you need lights. argued that the statute should
be only evidence of negligence. argued that the statute should be a prima facie case of
negligence.
i. Treating violation of statute as prima facie evidence shifts burden of coming forward w/
evidence to (but not burden of proof). If does nothing, judge will direct verdict on
standard of care, not causation.
ii. Jury says theres a statute, but violation doesnt mean they were negligent (TW
disagrees because they are likening statute to custom)
iii. Defendant tries to say that the violation was a prima facie evidence of negligence
1. Presumption of the plaintiffs negligence.
2. Plaintiff can introduce evident of non-negligence to rebut the presumption.
3. Plaintiff doesnt need to prove non-negligence, just present evidence. If so,
burden goes back to . If the doesnt rebut the presumption it stays in the
case. If you rebut the presumption you are saying theres some other standard
of care besides the statute.
c) Reque v. Milwaukee & Suburban Transport sued bus company b/c bus was too far from the
curb (statute setting regulation) and she fell when getting off bus.
iv. Court doesnt treat as negligence per se b/c the statute was designed to prevent other
risks (meant to keep roads open and not cause accidents with cars)
1. Statute is aimed at a class of people and must be in that group for negligence
per se to apply
v. Was there violation of a statute? Yes. Was the violation the cause of the harm? Yes. Was
the statute designed to prevent against this person and this harm? NO.

d) Three ways to treat violation of statute
i. Sets standard of care
ii. Treated as evidence of negligence (Twerski hates this; juries shouldnt be doing
statutory interpretation, its confusing. If judge wants to use statute, then use it!)
iii. Treated as a presumption (if can give info that he was acting reasonably, then the
presumption leaves the case, and they are back to the violins of telling their stories)
1. Twerski says Cardozo got it right in Herzog in saying unexcused failure to comply
with statute is negligence per se.
Page 20 of 60


e) Restatement, Third for Negligence per se
a. The court will not not adopt as the standard of conduct of a reasonable man the requirements of
a legislative enactment or an admin regulation whose purpose is found to be exclusively
i. To protect the state
ii. To secure people their rights as members of the public
iii. To impose upon the actor the performance of a service which the state undertakes to
give to the public
iv. To protect a class of people other than those who interests are invaded
v. To protect an interest other than that invaded
vi. To protect against other harm than which has resulted
vii. To protect against other hazards than from which the harm has resulted

f) Stachniewicz v. Mar-Cam Corp


a. Patron of a bar is suing bar owner for injuries inflicted by other customers in a bar fight.
b. Court considered the appropriateness of the standard and decides its valid and that
plaintiff was within class of persons that the statute was intended to protect.
g) Licensing Statute
a. Even harder to decide violations
b. Failure to obtain a license is not negligence per se.
c. NY statute says that the fact that a doctor would practice without a license is deemed a
prima facie of negligence.
i. Presumption of negligence that the actor has to present evidence
ii. Still need to prove common law negligence
h) Excused negligence per se:
i. Violation is reasonable because of actors incapacity
ii. Neither Knows nor should know of occasion for compliance (Twerski thinks this is
wrong)
iii. Unable after reasonable diligence or care to comply
iv. Confronted by an emergency not due to own misconduct
a. Compliance would involve greater risk of harm
b. Impson v Structural Metals, inc
i. Highway accident where three people were killed. Truck attempted to pass car
within a prohibited distance of a highway intersection. Car turned left into the
intersection and was struck by the truck.
a. Driver knew of the law and what he was doing so no excuse.
b. TWviolation of a statute has a generic quality
i. Negligence cases are fact sensitive
ii. You have to tell me fact sensitivity doesnt make sense to use the
statute in this case
iii. Tension between statute and ordinary litigation of a negligence case
v. Judge decides whether or not to use statute.
vi. ON EXAMyou see statute
a. Ask if it fits within justified excuse? Should statute be used?
b. Still have to see if statute creates a duty
vii. A statute sets a standard of care. If it doesnt set a standard of care it doesnt generally
belong in the case because it doesnt tell you how to act.
viii. HYPO 32 pg 205
ix. Actors who violate auto equipment statutes often use excuses.
a. Do these statutes set the standard of care? Justified excuse if he inspected car.
Page 21 of 60



F.) Res Ipsa Loquitur
DEFINITION:
a) WAKE FOREST LAW REVIEW ARTICLE
b) Cases where you dont know exactly what happened, but most logical explanation is that was
negligent when you consider all the things that could have gone wrong
c) The Thing Speaks for Itself
d) coming close to creating liability when there is no evidence
e) Restatement, Third, Res Ipsa Loquitor
i. the jury may infer that the has been neg when the accident causing the plaintiffs
physical harm is a type of accident that ordinarily happens as a result of the neg of a
class of actors of which the is the relevant member
f) More probably than not, was there negligence?
i. More probably than not, if there was negligence, are YOU the ?

ELEMENTS:
ii. occurrence doesnt ordinarly happen unless someone has been negligent (More
probably than not, theres negligence)
iii. Instrumentality that caused the injury was in s exclusive control
1. Twerski hates idea of exclusive control. Proposes standard: is there sufficient
evidence that points to the defendant as the cause of the harm
iv. Injury must not have been caused by

SCOPE:
a) can beat case by coming up w/ an alternative cause which stops the judge from giving case to
the jury. Since theres no evidence and just generalizations, should come up with evidence
that puts generalizations in doubt.
a. Say the generalization wasnt at work on this DAY. (bad storm)
b. Going at the heart of res ipsa because you have some evidence, not NO evidence that it
was the alternative case.

b) Singh v. UCP hit by motion sensor door- says there must be negligence. Court says its a jury
question against both s. Expect someone to squeal on the other . Twerski says res ipsa
shouldnt have been applied to this case.
c) HYPO 33 pg 214

d) Exclusive control has been expanded by courts
i. Ybarra v. Spangard - plaintiff woke up from surgery and had shoulder injury. He was
unconscious so he could not identify defendant that caused the harm
1. Court rules that all doctors/nurses have to explain their conduct regardless of
exclusive control
2. Dont know who is at fault but more likely than not it was the result of
negligence.
g) Procedural effects of Res Ipsa
i. Sullivan v Crabtree
1. Plaintiff is suing for damages when their son was killed in a truck accident when
he was a passenger in the truck
Page 22 of 60

2. Truck driver says many possible causes of accident.



e) Jurisdictions vary on the way they treat Res Ipsa/Procedural effects of Res Ipsa:
i. Inference of negligence - jury can believe or not believe it. ( should raise a shadow of a
doubt of liability)
ii. Presumption of negligence - requires the jury to find negligence if does not produce
evidence sufficient to rebut the presumption.
iii. Burden shifted to to prove non-negligence - It not only raises presumption but also
shifts the ultimate burden of proof to and requires him to prove by preponderance of
all the evidence that the injury was not caused by his negligence


Chapter 4: Actual Causation
A.) But-For Causation
DEFINITION:
a) has to show that harm wouldnt have happened if it wasnt for conduct (but-for test)
i. If the harm would have happened anyway, then is not liable
ii. Perkins v. Texas & N.O.R.-- train was going 37 mph when it should have been going 25
mph. Train collides with a car (cant determine how fast it was going; 3-25 mph). B/c it
could be shown that the accident would have happened even if the train wasnt
speeding (acting negligently), then RR is not the but-for cause. Judge directs for as
matter of law.
1. Needs to be a substantial factor of the harm
2. TWis this fair to the plaintiffs?
a. Using a hypothetical but-for
b. But you WERE speeding so why does the even have to answer the
hypothetical but-for?
ii. Ford v. Trident
1. Man overboard case
2. Improper safety equipment
3. How much cause is enough cause? Here, there was no cause.
b. Authors Dialogue 13 pg 228
iii. Jcourt was wrong in Ford. Shouldnt owner of boat lose?
iv. TWplaintiff didnt prove that the companys neg caused decedents death-he would
have drowned no matter how good their life-saving equip was
v. Jwithout the boat, he wouldnt have drowned. Boat was neg maintained
vi. TWneg maintenance didnt have to do with his death
vii. Jwhy should that matter? Imposing liability would help make him have better equip
1. Boat caused death
viii. TWwhat if the neg boat was in a horrible storm and everyone died? Equip didnt
contribute to harm
a. Burden of proof on but-for causation is on plaintiff
i. WEX MALONE ARTICLE
b) There can be more than one but- for causes
c) Cause-in-Fact partly a fact question and partly Policy :
i. Mere possibility that an injury could have happened without the negligence is
insufficient to break causal chain
Page 23 of 60

ii.

Reynolds v. Texas & Pacific Ry. fat woman fell down the stairs in an unlit stairwell.
was negligent in the safety of the stairs, but argues Mrs. Reynolds is an accident waiting
to happen.
a. Court holds if negligence of greatly multiplies the chances that will have an
accident, theyre not going to direct a verdict.
i. Twerskis problem with this: NEGLIGENCE ALWAYS INCREASES RISK OF
HARM.
ii. The mere possibility that she could have fallen without the negligent is
not sufficient to break the causal chainbut this wasnt a mere
possibility, it was more than that.
b. This case gets through on but-for b/c court cheats for policy reasons. We
want to protect the Mrs. Reynolds of the world.
c. Malonelook at case in its entirety to see if itll go to jury
i. Cant deal with cause in fact as an isolated question
d. **explain why it would go to judge or jury
e. HYPO 34 pg 229

d) Failure to Warn and Causation
i. If risk of harm is obvious, then no reason to warn
ii. If can prove that knew of danger from other sources, then no causation
iii. Heeding presumption
a. If P did not know and D did not warn, it is assumed that P would have heeded
the warning
b. Liability is therefore imposed for FTW in these cases
iv. Even when the doesnt know of the relevant risk, the negligent will escape liability
by proving that the would not have read the warning
v. When the risks arent obvious, but the proves that the knew of the danger, courts
have ruled as a matter of law that the s failure to warn didnt cause the accident

e) Considerations on the but-for rule (Malone article)
i. Degree of s fault (the worse the fault, the more likely that a court will let a case with
thin causation evidence go to a jury)
ii. breached a duty that was designed to protect against the very type of risk which
was exposed to (courts will send Q to jury even if little evidence)
iii. when the but for causation is absent, the will have failed to make out a crucial
element of the case and wont prevail
iv. actual causation is not just a factual question at to what would have happened if not
negligent
a. important policy concerns too
v. also if the breached a duty, the courts will be inclined to send the causation inquiry to
the jury
vi. but for causation is the proof, but the rule of going to the jury or not has to do with the
facts of the case

B.) Special Problems with Proof
a) Deciding whether s conduct itself had anything to do with causing harm
b) Use expert testimony to decide
c) Williams v Utica College
Page 24 of 60

a. College student assaulted in her dorm room. She


doesnt know if the assailant was someone who lived in
the dorm building or not. She sues the school for
negligence in security of the building
b. There is a duty here. Breach is a jury question
c. Causation as a matter of law
d. DUTY
i. Whether or not youre supposed to do anything
ii. foreseeability merely determines the scope of
the duty once the duty is determined to exist
iii. foreseeability goes to B<PL, standard of care,
NOT if whether you have a duty at all
e. CAUSATION
i. If hes an insider, the security is not the but-for
cause.
f. Martin v Herzog, Reynolds
i. Where the causal link btw the negligence and
the harm that occurs is strong, the jury can
decide, more probably than not, the injury
occurred because of the negligence
g. Has to be more probably than not increased the risk
d) Three factors that allow juries to find that a necessary but for not directly proven element for
liability was present
a. Knowledge if A is present, B is likely to happen
b. Greater capacity of the than the to explain what
happened
c. Finding error no liability is worse than finding liability
e) Kramer Service, Inc. v. Wilkins Guy injured when glass hits him in the head in a hotel. Hotel
admits to negligence for injury. Later he develops cancer in the injured spot. Sues for damages
related to cancer.
i. Not liable b/c medical community doesnt know what causes cancer. Possibility isnt
enough for causal link.

f) Use expert testimony to decide cases
i. Frye Test (General acceptance)
a. Before admitting expert testimony make sure it is sufficiently established to
have general acceptance in the field to which it belongs.
b. Used until 1995
ii. Daubert Test need more than general acceptance (heightens standard for letting
evidence in)
a. Plaintiff sees this as asbestos kind of thing
b. need to consider all parts of the method, and that it caused the harm, rather
than increased the likelihood
i. Factors to be considered:
a. Information has been or can be tested
b. Subjected to peer review
c. Potential rate of error
d. Has the opinion been widely accepted in the community
Page 25 of 60

e. Was this opinion formulated independently outside of the trial?


c. Twerski: thinks Daubert standards are appropriate. Dont want to let junk
science in. Jurors cant make sense of conflicting expert testimony. Judges are
better at screening out the evidence.
d. Remand of Daubert
i. Did it more than double the risk? Needs to be doubled.
ii. Scientists may set the level of proof to say A caused B at a very high
levelwho says courts should adopt the scientific level of proof for
damages? (Tension)
iii. General Acceptance test
a. Test for scientific validity
b. TW says this is bad for the because he has to prove four factors before he can
even use the expert testimony
iv. Causation: two questions
a. General causationcan this toxic agent cause the problem?
b. Specificdid it in fact cause the problem in this ?
c. Making out the general doesnt mean you made out the specific.
Authors Dialogue 14 pg 247
v. TW---Daubert-idea that scientific and technical expert testimony should meet min
standards of integrity seem self evident
vi. Jimwhy should we care that much whether expert testimony rigorously comports with
such standards?
a. The defs experts will make such weaknesses clear to the jury
b. Why should judges screen the purity of technical opinion evidence?
vii. TWyoure wrong. Juries cant really make sense of conflicting expert testimony. The
flesh and blood reasonable person construct doesnt work so well in high tech
situations. If we let plaintiffs rely on what the SC in Daubert referred as junk science,
snake oil experts for hire may do the jurys thinking for them.
a. Judges arent experts but better than jury
viii. Jim-but we do trial by jury
a. Daubert doctrine allows judges to throw out cases without them ever reaching
the jury
ix. Rider v Sandoz Pharmaceutical Corp.
a. Women suing Sandoz claiming the prenatal drug Parlodel caused them to have
strokes
b. Sandoz moved to suppress the expert testimony
c. Used Daubert test
d. In the absence of epidemiology, plaintiffs may still prove medical causation by
other evidencehavent provided reliable evidence to support their conclusions


g) Loss of Chance
i. In cases where a negligent misdiagnosis decreases percentage of survival, the courts
have imposed liability
ii. Herskovits v. Group Health doctor fails to timely diagnose patients cancer, his chance
of survival decreased from 39% to 25% and he dies. Wife sues for negligently reducing
his chance at survival
Page 26 of 60

a. If court goes by more probably than not test, then the case fails. Also not
cause-in-fact since but for the doctors negligence would have still died.
b. Twerskis issues: How do you measure your value of life? Should medical
history/ age be taken into consideration for recovery?
i. Is this a one-way street? You get 60% loss damages even if you proved
more probably than not.
c. Loosening the rules of causation for policy reasons- we dont want doctors to
get off the hook for not diagnosing illnesses that are lower than 50% survival.
iii. Majority of jurisdictions allow loss-of-chance claims to go to the jury even when
cannot prove that was, more likely than not, the cause of s harm ( would have died
anyways)
iv. King theory
a. Allow dollar amount to reflect loss of chance
v. Twerski Questions
a. HYPO can prove more probably than not 60 to 40 chance of survival
i. wont want proportional recovery
b. Does the have a chance to go after the and say you didnt have a 39%
chance of recovery, thats the national average. You have family history.
i. Dont know.
c. How far can you push the loss chance idea?
i. You can push it to an extreme
d. Restatement, third
i. The lost-opportunity development has been halting, as courts have
sought to find appropriate limits for this reconceptualization of legally
cognizable harm

C.) When Two (or more) Negligent Actors Cause Harm
a) YOU DONT GET OFF THE HOOK BY POINTING TO SOMEONE ELSE (can have countless but-for
causes)
b) Where separate acts of negligence combine to directly produce a single injury, each tortfeasor is
responsible for the entire result, even though his act alone might not have caused it.
c) Joint and Several Liability-
i. s who are jointly liable can be joined in a single action (though they dont have to be).
s who are severally liable are each liable for the full damages, although is entitled to
only one total recovery.
a. Arises from: 1) s acting in concert and 2) acted independently by caused
indivisible harm
ii. Hill v Edmonds
a. Plaintiff is a passenger injured in a car accident btw the negligent driver and
negligent truck owner. Truck driver is negligent because he left his truck,
without lights, in the middle of the road. Car driver is liable because she ran into
the truck.
b. Each tort feasor is responsible, joint tortfeasors
c. The negligent conduct of either would not, without the other, have casued the
accident in which the plaintiff suffered harm
iii. Kingston v. Chicago Two fires -1 caused by sparks from s train. The other originated
from an unknown source but conclude that it wasnt a natural fire, instead it was started
by a human (responsible origin)
Page 27 of 60

a. RR is responsible for 100% of the damages b/c the two fires are jointly and
severally liable. Each sufficient to cause damage.
b. Court uses Restatement, 2nd 431 idea of substantial factor to solve the 2 fire
problem - an actors negligent conduct is a legal cause of harm of another if
his conduct is a substantial factor in bringing about the harm
c. Substantial factor was confusing, so now Restatement, 3rd 27 describes
Multiple Sufficient Causes If multiple acts occur, each which would have been
a factual cause of the physical harm in absence of the other, each is regarded as
the factual cause of the harm
Authors Dialogue 15 pg 259
iv. Jim: natural origin fire issue in Kingston
a. If we assume that most fires that are not shown to have been caused by the def
(or some other train) will be of unknown origin, and if Kingston assigns
responsibility for all such fires to the def, then Kingston accomplishes the same
result of holding the train liable for natural origin fire as does Anderson and the
Restatement. And at least these sources are candid about it
v. TW: Kingston says that the def train will be liable whenever the unknown fire is of
responsible origin. Then it equates responsible with human. I assume that if the
unknown fire had been shown to have been started by a couple of non neg, judgment
proof little kids, the court would have said they saw that as a responsible origin and
would have found the train liable even though the kids are basically natural causes. Even
tort law doesnt think young children can be neg.
vi. HYPO 35 pg 260


D.) When One of Several Negligent Actors Clearly Harmed , But Dont Know Who
a) When you cant tell which of the 2 caused the harm, hold them both liable, and leave it up to
the Ds to figure out which one was innocent
i. Summers v. Tice Two s both negligently shot in the direction of , one bullet hit s
lip, the other hit s eye. Cant determine which bullet came from which guy.
a. court says one of you did it, so well try you as joint tortfeasors unless you can
prove otherwise shifts burden of proof from to
i. Dont want to to not be able to recover b/c he cant say definitively
who did it (imposing liability on actors who we know didnt actually
cause the s harm, which seems unjust)
b. Ybarraharder because 5 or 6 people might not have been negligent
ii. HYPO 36 pg 262

b) Market Share Liability
i. Sindell v. Abbot Laboratories develops bladder tumor by drug DES that 200
manufacturers made. One company supplied it to s mom, who took it while pregnant,
causing cancer in daughter. sued 5 of the 200 manufacturers; 20 years after the mom
took the drug. She was unable to determine which manufacturer produced her mothers
DES, if any.
a. Court applies market share liability so that can recover. Held each
responsible for the % of the market that it controlled
i. can prove its way out of liability (ex: one manufacturer stopped selling
drug before mom was pregnant)
Page 28 of 60

ii.

iii.
iv.
v.
vi.

vii.
viii.

ii. Courts are VERY hesitant to use doctrine (DES was very unique cases-
fungible product, causation was definite)
iii. Why not shift burden of proof to like in Summer?

a. Same drug for everyone
b. Court not ready to pick one for proof because that one gets
huge liability if they cant meet the burden.
c. Should joint civil liabilities apply?
i. Ex. 5 defs make up 50% of market, should they
all have to double liability? TW says no and so
does Restatement.
Twerskis issues with market share
a. The transaction costs of proving yourself out may be overly
burdensome/unfair/lead companies to settle and pay rather than pay costs of
litigation.
b. How do you define the market? (national, state, city? Joes pharmacy?) Court
decides to use a national market. NY rule says you cant prove yourself out.
(Twerski thinks they got it right)
c. A lot of states dont use market share because it violates causation.
d. Market share has showed up in asbestos brake lining cases
i. Different from pill because not identical
ii. Trying to do market share is close to impossible
Authors Dialogue 16 pg 271
JIM: market share-interesting idea but it wont have lasting impact
TWpowerful impact on DES cases
JimDES, a generic drug with the same dosage utilized by women for a short period of
time during their pregnancyinjuries were unique
a. Attempts to make market share outside DES has failed
b. Most court have refused to apply it to auto asbestos cases
i. Mechanics are suing auto manufacturers for exposure to asbestos in
brake linings and getting cancer years later
ii. Market share shouldnt be applied because the asbestos content and
nature of the exposures differ so radically from one type of brake lining
to another
iii. Some asbestos give off toxic matter some dont
iv. Time span of exposures vary
v. How could courts use a market share in these circumstances?
TWmust ask, yeah but is it fair? Assume auto manufacturers were neg in using
asbestos in the brake linings. Is it fair to deny workers who developed cancer a recovery
against the auto manufacturers as a group because we cant figure out the % of blame?
Jimyoure wrong. We dont know if any given manufacturer was responsible.

Chapter 5: Proximate Cause

A.) Proximate Cause


a) Generally
i. Once you reach proximate cause, you have already established 1) Duty 2) Breach of Duty 3)
Cause- in- Fact
ii. Proximate cause is a second screen to limit liability
Page 29 of 60


DEFINITION:
a) Jury must decide whether risk caused by the Ds negligence falls within the scope of the foreseeable
risk created by his conduct
i. If the risk was not reasonably foreseeable (within the scope of the risk), then D is not liable
a. Twerski talks about bundle of risks or worry list
ii. We dont use B<PL for the particular risk because P would more than likely be very low and
wed rarely find negligence
iii. If s conduct was offensive and the harm was very bad (i.e. beyond mere negligence), the
scope of risk is read broadly, and most likely will be held liable.

b) Restatement, Third 29 Limitations on Liability for Tortious Conduct
i. states that a jury should be told that, in deciding whether s harm is within the scope of
liability, it should go back to the reasons for finding the engaged in negligent or other
tortious conduct. If the harm risked by that tortuous conduct includes the general sort of
harm suffered, then D is subject to liability.
a. Courts never actually give this instruction to a jury. (Twerski: juries are told
gibberish)
1. Reason is we dont want juries to focus on the particular risk
b. Twerski: This is a good instruction to aid a judge in figuring out whether or not
to direct verdict. Jury should be told, when deciding if the harm is within the
scope of liability, see if risks are in the general sort of harm suffered by . Risk
standard.
c. Takes foreseeability out of duty question
d. Essential role of policy considerations in the determination of scope of liability
1. Scope of liability is fact intensive (LAW REVIEW ARTICLE)
2. Foreseeability is still relevant in scope of liability question, not duty question
ii. Similar view in Marshall and reflected in Thompson, Gibson
iii. Foreseeability belongs in:
a. Breach of standard of care which is B<PL
1. In deciding if case will go to jury, judge will decide 2-8 question. If outside
that, direct verdict.
b. Once past standard of care, foreseeability comes into play in proximate cause
and risk list
1. Look at foreseeability in terms of the original negligence
2. Its a common sense question
iv. In negligence per se, legis intent is essentially risk list question
c) CARDOZO View to approach negligence
i. Do it using duty language
ii. Went about the process backwards
iii. Thompson says protect jury
d) Andrews view
i. Scope of risk and foreseeability can all come into play but thats a matter of practical politics
ii. Refuses to be bound

e) Arbitrary limits on Foreseeability
i. Some courts impose arbitrary limits on what is and is not foreseeable
Page 30 of 60

a. Ex: New York fire rule liability for negligently caused fire on s premises
extends to the next property (whether or not its adjoining) and no further

B.) Liability Limited to Reasonably Foreseeable Consequences
a) Perkins recap
i. Train going 37mph until city limits, then goes 25mph
ii. Was the negligent? Yes, he was speeding
iii. Was the negligence the but for cause? Yes
iv. TWproximate cause should be called XYZ
v. If hes the but for cause, why not liable? Not fair to hold him liable. What took place is not a
risk anyone would have contemplated happening
vi. The risks that made the conduct tortious are unrelated to the harm that happened
b) Marshall v. Nugent - Truck and car crashes into e/o, no one is hurt, truck driver suggests someone
should walk up the hill and warn others about the stopped traffic. Marshall goes up the hill and is hit
by another car. Have to decide if truck is the proximate, legal cause of the later accident.
i. Rule the harm caused is in the bundle of risks originally foreseen in the negligence
(injury from oncoming traffic), so proximate cause is a jury issue
ii. If we already did B<PL, why do we need to ask the foreseeability question twice?
iii. Dont attempt to fragment or they wont be negligent when they are.
iv. Ask bundle of risks then ask if THIS harm was within the bundle of risks (scope)
v. Judge can direct verdict and has to decide where risk list ends. If youre clearly on one side
of the list, its a directed verdict. If not, its the jury.

c) Demers v. Rosa let her dog roam in the streets. Police get a call to get the dog back. Its
snowing/ sleeting outside. Police retrieve the dog, put it in the car, and as hes walking on the
driveway, he slips and falls.
i. Rule Not within the bundle of risks that are foreseeable when negligently lets her dog
roam.
a. Twerski: once you have facts of the case and negligence established, its really a
policy question how far youre going to push scope of the risks
b. Only way to do proximate cause analysis with a statute is to figure out the
legislative purpose for the risk list.
d) Authors Dialogue 17 pg 287
i. TW: the cases Bunting and Daniels suggest that in some cases unforeseeability of outcome is
no problem under the prox cause right? Even rare events are included within the prox
cause?
ii. Jim: Yes, at least regarding the details of what happens to the plaintiff. Anything that
doesnt seriously affect the type or magnitude of the plaintiffs harm is a detail. Details
concern not WHAT happened, but HOW it happens. The risk was the explosion, the rat was
a detail
iii. TW: Driving neg and hit an NBA player. Am I liable for him being out for the season?
iv. Jim: Yes.
v. TW: So NBA is a detail????? Weird!

e) Thin Skull Rule you take the as you find him, is not entitled to a healthy , may be liable in
damages for aggravating a preexisting condition)
i. Also applies to:
a. property damage
Page 31 of 60

ii.

iii.

1. Polemis (English case, 1921)


a. As long as negligent act directly links to the harm done, thin skull
applies. dropped a plank, could have foreseen damage to ship,
didnt have to foresee that it would cause an explosion
b. fragile psyches
1. when negligently caused physical invasion triggers unusual and debilitating
psychological response
DOES NOT apply to:
a. s who suffer special injuries as a result of religious beliefs or past mental
trauma
1. Ex: couldnt recover after she was in a car accident and refused corrective
surgery for religious beliefs and was confined in a wheelchair as a result (if
she had had the surgery, she wouldnt have been in the wheelchair)

McCahill v. NY Trans is hit by s taxi and injured. Two days later dies in the hospital
from delirium tremens (but was alcoholic).
a. Rule - Negligent person is responsible for the direct effects of his acts, it doesn't
matter that the facts are somewhat unusual or if had a prior disorder.
1. argues the was going to die anyway b/c he was an alcoholic this
argument can be used to lower damages, but not as a defense to escape
liability

iv.

f)

g)
h)

i)

JIM: a def need not take into account the idiosyncrasies of possible plaintiffs when acting.
She is entitled to assume its a person of ordinary capacity.
a. Does this fit in what weve said about the scope of risk?
v. TW: I dont know who Im going to hit
a. Scope of risk is very broad with regards to who Im going to hit
b. Have to set standard of care with all of them in mind
c. thin skull rule makes sense within scope of neg

Negligence Per Se and Proximate Cause
i. If the damage is not the sort of risk that the statute was intended to protect against, the
prox cause link between negligent conduct and s harm is missing.
a. When relevant, safety statutes should cut off dispute about duty and breach
b. Twerski basically doing the same thing, just go back to legislative intent and
come up with their risk list
HYPO 37-38 pg 295-96
Authors Dialogue 18 pg 297
i. TWthe s harm must be the sort the legis had in mind when the s conduct was judged
to be negligent
ii. Jcant the legis have more than one purpose in mind?
iii. TWthe court may say the act was passed for only one purpose
iv. Jhow does the court know what every member of congress who voted for it thought
about its purposes?
v. TWwhere relevant, safety statutes should cut off disputes about duty and breach.

Palsgraf v. LIRR Palsgraf is standing on the platform waiting for a train. Man runs to get on the
train, LIRR guard helps him on; man drops a box that has fireworks in it (though no one knows). The
explosion causes scales to fly from the other end of the platform and hit/ injure Palsgraf.
Page 32 of 60

i.

Cardozo cuts off the case by saying theres NO DUTY to Palsgraf


a. Twerski: Of course they had a duty of care to Palsgraf! Common carriers have
duty to their passengers. Cardozo shouldnt have isolated the risk (should have
considered the general risks)
b. Cardozo knows what hes doing. Hes cutting the case for policy reasons.
1. He doesnt want to give proximate cause question to the jury. Cardozo
suggests that wasnt negligent (its not their responsibility to inspect every
package)
2. Twerski: Negligence might have been moving with the doors open. But, if
this turned into a manner of harm case (the HOW), then it would have gone
to a jury
a. Twerski: foreseeability of the harm doesnt belong in duty analysis,
it belongs in risk/utility analysis
b. AT: should have gone to a jury on the scope of the risk
c. TWhe should have gone through the whole negligence analysis instead of
stopping at duty.
ii. Dissent (Andrews)
a. There was a duty owed to Palsgraf. He defines prox cause not only by scope of
the risk, but by a rough sense of justice (in taking policy into account too)
1. Twerski: Andrews also got it wrong. You have to look at the scope of the risk
first!

j) Thompson v. Kaczinski s left parts of a trampoline disassembled on their prop. A storm comes
and takes the parts onto a nearby highway, where has to swerve to avoid them and crashes.
i. Rule foreseeability shouldnt be considered in a duty question. Duty is a policy issue.
a. Once you get past duty, look to negligence and if the harm was within the
bundle of risks
b. If there wasnt a breach of duty or the s conduct wasnt the proximate cause,
then is not liable
c. Restatement, Thirdan actor has a duty to exercise reasonable care when the
actors conduct creates a risk of physical harm
ii. In extraordinary cases, the general duty may be replaced by something else for policy
reasons
iii. Courts should leave standard of care and breach to jury unless its really obvious
iv. Restatement, second
a. An actors conduct is negligent if
1. His conduct is a substantial factor in bringing about harm (third changes this
and does risk standard)
2. There is no rule of law relieving him from liability
k) Policy reason for no duty in certain class of cases?
i. TW agrees but not that it has to be in a certain class of cases
ii. Ruling should be explained by articulating a policy reason for no duty of reasonable care
l) Authors dialogue 19 pg 314
i. Jwhy was Andrews compelled to say in his dissent in Palsgraf it is all a matter of
expediency?
a. I hope it isnt true. Expediency implies to me that judges are free to reach any
outcome that suits them. But theyre not. They have taken an oath to uphold
the law
Page 33 of 60

ii.

iii.
iv.

TWbut doesnt the law of prox cause allow judges to reach either outcome most of the
time? Couldnt they write a reas opinion to support a decision for the or the in most
cases?
Jby allowing them to reach either outcome, you are clearly correct if you mean they wont
be impeached. But youre wrong if you mean they should feel free to reach either outcome.
TWa judge could be influenced by bias.

SCOPE:
a) Things to consider:
i. The WHO was the particular foreseeable
ii. The WHAT was the injury/ harm foreseeable
iii. The HOW was the manner foreseeable
a. If the answer is yes to WHO and WHAT, question will almost always going to go
to a jury. Wont have a directed verdict if only the HOW wasnt foreseeable
1. Ex: Buffalo River Crisis barge breaks loose, crashes into another barge, it
damns up the river and leads to major flooding. The and the harm was
foreseeable, the way it happened wasnt foreseeable. Going to send to the
jury
b. Twerski: thinks the HOW does matter in determining liability (Ex: Twerskis
driving on Joralemon and hits a pedestrian who turns out to be Shaq, doesnt
think he should be liable for millions for wrecking his career)

C.) Superseding Causes
DEFINITION:
a) An act that occurs AFTER s negligence and contributes to causing s injury
i. Twerski: this is really just another level of proximate cause analysis
a. Was the intervening/superseding cause foreseeable? Was it within the scope of
the risk of s conduct?
1. If so, then still liable
2. If not, then under proximate cause D is not liable for his conduct

b) Restatement, Third, 34:
i. When a force of nature or an independent act is also a factual cause of physical harm, an
actors liability is limited to those harms that result from the risks that made the actors
conduct tortious
a. Shifting full liability to an intervening party is rarely justified

c) Derdiarian v. Felix Contracting Corp. Construction site didnt have the right safety barricade, an
epileptic driver has a seizure on the road, crashes through the site and causes injury to (he ignited
into a fire ball from a gas spill)
i. negligent in not putting up the right safety guards
ii. was foreseeable and harm was foreseeable, only question is the HOW goes to a jury
iii. Rule Where the acts of a third person intervene between the s conduct and the s
injury, the causal connection is not automatically severed
iv. court directed verdict for (TW says jury)
d) Barry v. Quality Steel - scaffolding collapsed with s on it. Scaffolding and brackets were made by
Quality. The brackets may have been wrong size for the scaffolding. DeLuca, construction company,
Page 34 of 60

removed a supporting scaffold. One employee negligently attached brackets of scaffold to roof.
Two potential intervening/ superseding causes
i. Rule Get rid of superseding/ intervening cause instruction for juries. It just confuses them.
Use simple question was it within the scope of the risks
a. Court says we should just apportion fault along the lines of comparative fault.
Dont need to do all or nothing.

SCOPE:
a) Acts of Nature:
i. If put in a position where was then harmed by an act of nature, (lightening, floods,
avalanche) courts usually hold that is not liable.
b) HYPO 29 pg 325
c) Intentional Criminal Actions:
i. If unforeseeable intervening act is a crime or intentional tort, it will usually cut off liability
ii. Watson v. KY & IN Bridge s let gas escape from railroad car, created pools of gas. Guy
walks by and drops a match. suffers damage from explosion. Is still responsible?
a. Rule liability depends on whether the guy threw the match on purpose or if it
was an accident.
1. HOW case, but when the intervening cause is a criminal, its not within the
scope of the risks and the is no longer liable
2. Twerski when human agency comes into the situation, its no longer just a
detail
i. Theres something unfair about letting an intentional tortfeasor
walk away
ii. Considering policy criminal acts might be foreseeable, but we
dont want them to get off the hook
3. Theres a final check after the whole analysis. Ask, does this make sense?
iii. Authors Dialogue 20 pg 328
iv. JIM: court got it wrong. Why does it matter how he threw the match and his intent?
a. His state of mind should be a detail
v. TW: his willful act comes in big time, not like the detail of the NBA player.
vi. Jim: A sparrow flies over the gas, grabs a piece of it and causes an explosion. Should the gas
spiller be liable?
vii. TW: yes. The sparrow are details. The match thrower is different and takes charge of the
situation
viii. JIM: it became his and the defs fire. The both should be liable jointly
ix. TW: I would agree if the jury instruction asks if throwing match was foresee
x. JIM: why does it mater if it was neg or reck?
xi. TW: relevant, but not controlling
xii. JIM: while details may be relevant under general foreseeability, they shouldnt control the
oucome
xiii. TW: his state of mind isnt a detail. Unfair about holding def liable when the real cause of
the injury is a deliberate criminal act

d) Intentional Acts after caused harm to
i. Fuller v. Preis guy gets into a car accident, suffers some brain damage. He gets really sick
and loses the will to live. Ends up committing suicide. Leaves note for his family saying hes
sane and knows what hes doing. Family sues negligent driver who caused accident.
Page 35 of 60

a. Rule - There was enough evidence the suicide was a proximate result of the car
accident to at least send the issue to a jury.
1. Suicide is not outside the scope of the risks as a matter of law
2. There are limits to liability though (motorcycle accident victim is paralyzed,
asks his brother to kill him, couldnt recover for wrongful death from
manufacturer that caused the defect in the bike)
3. liable if suicide is irresistible impulse
i. TW says it doesnt have to be irresistible, just within range of
foreseeability

D.) Duty/ Rescue Rules
a) Wagner v. International Railway - s cousin fell off train ( was negligent b/c didnt close the
doors). went to go look for him with search party and fell off a bridge.
i. Rule - danger invites rescue, b/s is liable for the risk they create from their negligence,
they are liable for the risks of a rescue situation
a. Rescue was foreseeable. was foreseeable. Reasonableness of the is an issue
for the jury (separate question from proximate cause)
b. HYPO 40 pg 338


SCOPE:
a) Applies when:
i. If is injured during a rescue that he thought was necessary, but it turns out it wasnt, he
can only recover if its found that reasonably believed someone was in need of rescue and
s actions were reasonable
b) Doesnt apply when:
i. comes to the scene for purposes other than rescue (i.e. direct traffic, came to the scene
after the accident happened didnt intervene in anyway)
i. TW- Why do they have to be a rescuer to be within scope of
risk?

c) Hamilton v. Beretta - 7 s brought negligent marketing claim against 25 handgun manufacturers
who sold guns that got into the black market and led to high crime/injuries. Issue whether owed
s a duty to exercise reasonable care in the marketing and distribution of the handguns they
manufacture.
i. Rule - Scope of duty determined by foreseeability; must show that owed a specific
duty to him or her.
a. Court doesnt get to prox cause issue b/c its too complicated to determine. Cuts
off the case with no duty.
ii. Policy concerns - court didnt want to extend liability to 3rd parties. Duty has to be to
specific , not just to society as a whole
iii. Market share liability not going to apply MS to this case b/c guns are not identically
interchangeable like DES and the negligence differs from company to company
iv. Proposals to stop this problem are hard to implement
v. How do I know the gun that shot the came from neg marketing or non neg marketing? I
have to know its from neg marketing.could use proportionality case but thats hard ot
find %
vi. Cause in fact issue is a nightmare then STILL have to do market share
Page 36 of 60

vii.

To decide duty you have to look at everything. Hard to deal with these mega problems
(asbestos).
Authors Dialogue pg 346
d) Jim-playing the duty card shares a common elementplaintiffs want the courts to require defs to
act as watchdogs to protect society from the violent or antisocial behavior of criminal actors. And I
could not agree more with the courts response to such requests of NO
e) TW: dont be so quick to dismiss the possibility that the defs in these cases should bear some of the
responsibility for what happens to the victims. In cases like Hamiltonit relates to a bartender
continuing to serve a drunk person and letting him drive.
f) Jim-but what about social guests? Do they have to watch over drinkers? I dont think so. The scope
of potential responsibility is too broad and fuzzy. And cases like Hamilton strike me the same way.
We cant ask gun manufacturers to be watchdogs.
g) TW-it depends on the circumstances of each case
h) Jim-I dont think we should impose liability, so long as there is any legit purpose served by having
such ammunition available to the public.
i) TW- you didnt like the Watson case, in which an arsonist behavior cut off completely liability as a
matter of law in an action against the railroad that neg spilled gas. Why should the railroad in
Watson have to pay for an arson while the gun maker in Hamilton escapes liability? Inconsistent?
j) Jim-good point if the right was to read Watson is that the arsonist, not the railroad, is somehow the
real bad guy. I tend to see tort liability as connected with ordinary notions of moral responsibility.

k) Duty rules dont usually apply:
i. to social hosts who serve their guests alcohol
a. Exception if they have specific knowledge that guest is drunk and let them drive
ii. to passengers in a car when they know the driver is drunk
l) Robert Keeton
i. A negligent actor is legally responsible for the harm, and only the harm, that not only (1) is
caused in fact by his negligent conduct but also (2) is a result within the scope of the risks by
reason of which the actor is found to be negligent.

Chapter 6: Limited Duty Rules
DEFINITION

a) Cuts off s liability even though hes acted badly and the s harm was reasonably foreseeable
A.) Limited Duty
a) In general, theres no duty to rescue strangers in American law
b) Restatement, Third 37:
i. Actor owes no duty to rescue when he didnt create the risk
ii. SOME exceptions when theres a preexisting relationship between person in danger and
potential rescuer (like common carrier/ passenger; school/student; employer/ employee)
a. Restatement, Third 42. Duty Based on Undertaking
1. You have a duty to act with due care once you undertake rescue
operations or if youve increased the risk of harm by deterring others to
rescue (making it worse has to be cause in fact)
2. The person to whom the services are rendered or another relies on the
actors exercising reas care in the undertaking (must mean Im warding
others off)
Page 37 of 60

3. (when you get involved, you cant get uninvolved)


4. TW: have to ask cause in fact question anyways. Breach of standard of
care doesnt get you homehave to go through negligence factors.

c) Yania v. Bigan came onto s property to discuss business. was mining coal and there was a
big trench with water. asked to help him start the pump and encouraged him to jump in. He did,
and drowned. Wife sues for wrongful death.
i. Rule No duty to warn of an obvious danger and no duty to rescue him. Court says taunting
doesnt change the duty here ( was an adult in control of his mental faculties)

a. Twerski: not sure that there was no duty to rescue. was business invitee (class
is owed general duty of care). Its hard to prove causation in rescue cases
though. Ex: could have thrown him a rope and he still might have drowned.
1. He said this would go the other way today. was the reason for the
harm and he IS the potential rescuer. He could have done an easy
rescue. This is not a case with 10 potential rescuers, he IS the rescuer.

d) Baker v. Fenneman and Brown
e) Invitees Baker guy went into Taco Bell, passed out, hit his head/ was knocked unconscious.
Cashier says she asked him if he needed an ambulance. He said he was ok, but then fell again. sues
Taco Bell for breaching its duty to aid. Court says Taco bell had a duty to provide reasonable care.
i. Rule Duty of reasonable care owed to someone whos invited into a business
establishment
a. duty is a balance between the relationship of the parties, foreseeability and
public policy concerns
b. Twerskis issue the 2nd fall was right after the 1st, even if the cashier had called
911, they wouldnt have gotten there in time. Ppl who work at TB arent
paramedics, theyre not going to help much Cause in fact question

f) Employer/employees Stockberger - Prison employee was insulin-dependent diabetic. He had an
episode and his co-worker offered him Ensure but then P was insisted on going home. Employer did
not provide a ride or restrain him; P got into a car accident and was killed.
i. Rule Court says no duty; didnt see the special relationship once worker left his place of
employment and wanted to. Could lead to false imprisonment issues.
a. At this point, IN didnt have duty liability to invitees
b. Did the co-workers start a rescue by giving him Ensure?
c. Twerski: hard to tell when a rescue was started and abandoned
g) Restatement 314A: Special Relations Giving Rise to Duty to Aid or Protect
i. A common carrier is under a duty to its passengers to take reasonable action:
a. To protect them against unreas risk of physical harm and
b. To give them first aid after it knows or has reason to know that they are ill or
injured, and to care for them until they can be cared for by others
ii. An innkeeper is under similar duty to guests
iii. A possessor of land who holds it open to the public is under a similar duty to members of
the public who enter his premises upon invitation
iv. One who is required by law to take or who voluntarily takes the custody of another under
circumstances such as to deprive the other of his normal opportunities is under a similar
duty to the other
h) Third Restatement
Page 38 of 60

i.

To give aid to one who is ill or injured extends to cases where the illness or injury is due to
natural causes, to pure accident, to the acts of third person, or to the neg of the plaintiff
himself, as where a passenger has injured himself by being clumsy

i) Stockberger v US
i. Diabetic not stopped from driving by his coworker and dies
ii. Posner said the employees had no duty to stop him.
iii. They gave him Ensure (made him think he was better) so they could be participants
iv. TW: not sure if this is right
j) HYPO 41 pg 368
k) Gipson v. Casey
i. Work party. gives woman drugs, knowing she will be giving it to her boyfriend who had
been drinking. He dies.
ii. New Rule: foreseeability is NOT a factor when determining if there is a duty
a. TW agrees. It belongs in standard of care and prox cause.
iii. Public policy: we are better served when we have duties to help prevent future harm
a. The fact that theres a statute for this means theres significant public policy
against it (not using statute as standard of care).
l) Twerski Law Review Article Violin and Cleaver
i. Restatement, Third, Liability for Physical Harm
a. Got it right
b. There are two diff legal doctrines for no liability: no duty and scope of liability
c. No duty=court and scope = jury
ii. Restatement, Third, Duty
a. An actor ordinarily has a duty of reas care when the actors conduct creates a
risk of physical harm
b. In exceptional cases (policy)...court can decide no duty
c. Comment says it is okay for courts to consider conflicts with social norms in
deciding duty
d. Unless a court can identify policy concerns applying to a broad range of cases
that can be categorized, they should not resort to a no duty analysis
e. No single category can describe the scope of the no duty rule..no substitute for
analysis of facts of case!
m) Special Relationship (Husband/ wife) J.S. and M.S. wife had reasonable cause to believe that her
husband was sexually abusing two young girls. Theres a NJ criminal statute that provides a duty to
report when someone reasonably suspect abuse.
i. Rule Court imposed duty in this case, based on the specific facts. In general, no duty
though.
a. just b/c theres a criminal statute imposing duty, doesnt mean it has to be used
in civil (TX court in Perry decided not going to use crim. offense as duty
standard)
b. Twerski: *** Again, foreseeability shouldnt be in duty analysis (creates wide
discretion to give cases to juries). Case is very fact specific, probably wouldnt
put duty on a neighbor who had suspicions this was going on. This sounds like
Andrews (Palsgraf), kitchen sink for duty.
n) Always ask if the statute sets the standard of care!
o) HYPO 42 pg 375

Page 39 of 60

p) Special Relationship (Psychiatrist/ potential victims) Tarasoff patient tells his psychologist he
wants to kill Tarasoff. Psychologist reports to the campus police, they detain patient for some time
and then let him go. A little while later, patient kills Tarasoff. Parents sue for breach of duty in
psychologist not warning Tarasoff.
i. Majority Rule in this country- Once a therapist determines or reasonably should have
determined (known or should have known) under professional standards that a patient
poses a serious danger of violence to others, he bears a duty to exercise reasonable care to
protect the foreseeable victim of that danger.
a. Twerski- didnt have duty to Tarasoff b/c there was no special relationship
between them. Putting duty to warn in these cases limits the trust in the
profession
ii. HYPO 43 pg. 384
q) Authors Dialogue 22 pg 382
i. TW: The no duty to rescue material bothers me. Do the courts really mean that I am not
liable if I dont rescue someone I could have easily rescued?
ii. J: I think they are correct in denying liability. Long run consideration of fairness and
workability must take precedence over short run considerations. You assert that there is no
cost to the rescuer? What if he has a fear of water?
iii. TW: thats subjective and the traditional negligence standard is objective. What a normal,
reasonable person would have done.

B.) Economic Loss
a) In general, D is not held liable for pure economic loss in the absence of physical injury/damage
i. Concerned with unending liability (where do you cut it off?)
ii. Economic loss should be handled by non-judiciary remedies (like insurance and contracts)
b) Exceptions
i. If s are part of an identifiable class that s knows or can foresee will be harmed, then
liability may be imposed
ii. If s have Quasi property interest (fish to fisherman)
a. State of Louisiana - Two ships crashed, one spilled PCP everywhere, Gulf closed
for several weeks. Several s sued the ship owners ranging from restaurants to
fishermen
1. Rule - Generally, no recovery for pure economic loss in the absence of
physical damage to a property interest, but some people who had
quasi property that was damaged could recover all the fish that
fisherman caught and sold were killed. Judge decides.
2. Twerski Doesnt like quasi-property interest exception; thinks there
should just be a bright line rule either for or against no recovery on pure
economic loss.
iii. HYPOs 44-46 pg 396
iv. People Express Airlines
a. Airlines is suing for neg that caused a dangerous chemical to escape from a
railway tank car, resulting in the evacuation of people, including airline
passengers. Disrupted business, econ damages.
b. DEFENDANT OWES A DUTY OF CARE TO TAKE REASONABLE MEASURES TO
AVOID THE RISK OF CAUSING ECONOMIC DAMAGES, ASIDE FROM PHYSICAL
INJURY, TO PARTICULAR PLAINTIFFS COMPRISING AN IDENTIFIABLE CLASS WITH
Page 40 of 60

RESPECT TO WHOM DEF KNOWS OR HAS REASON TO KNOW ARE LIKELY TO


SUFFER SUCH DAMAGES FROM ITS CONDUCT
c. An identifiable class is not just a foreseeable class (everything is foreseeable)
d. Plaintiffs must be particularly foreseeable class
1. Determine whether the econ injury was prox caused and determine if
theres a duty
e. Plaintiff who has breached his duty of care to avoid risk of econ loss to
particularly foreseeable plaintiffs may be liable for econ losses that are prox
caused by it
f. Airline had quasi property interest in the operation of the terminal
v. Authors Dialogue 23 pg 402
a. JimI think the reason courts dont recognize tort claims for pure econ loss is
that, from the broader, social welfare perspective, some forms of pure econ loss
arent really losses at all
b. TWso if I have to shut down my deli its a real loss to me though
c. Jimnot a loss to society. Deli down the block gets the money. Transfer of
wealth rather than destruction of wealth. Society doesnt lose
d. TW-I should sue the other deli?
e. Jim-no. neg guy who closed you down didnt destroy your profits, so much as he
moved it to other deli
f. TW-what if he burned me out?
g. Jim-hed have to pay for property damages but you dont have a property
interest in the expectation of future profits, apart from actual deli property
h. TW-why do fishermen recover when fishing grounds are damaged? Dont own
fish until they catch them
i. Jim-quasi property interest in fish same with airline in People Expressquasi
property interest in operation of terminal
j. TW-no

C.) Emotional Distress
a) The Impact Rule
i. Originally, the law was there could be no recovery for fright alone (or physical
manifestations of fright) w/o impact
a. Has generally been overturned in most jurisdictions
b) Zone of Danger Rule and Exceptions
i. can recover damages when he
a. Is in danger of physical impact
b. Reasonably fears for his own safety and
c. Suffers serious emotional distress b/c of that fear
ii. Restatement, Third 46 Negligent conduct directly inflicting motional disturbance on
another (dont have to have physical impact):
a. s negligence placed you in immediate danger of bodily harm; OR
b. fits into a class previously established by courts (exceptions to allowing
recovery without immediate danger of bodily harm)
1. Corpses, genetic material, special relationships (hospital to patients
daughter; some contractual relationships), etc. still need emotional
distress as a result
c. HYPO 47 pg 411
Page 41 of 60

iii.
iv.

TW: Zone of danger is a duty rule, if not in the zone, no duty


Courts step in a big way and cut off a lot of these cases

c) Physical Manifestation Rule:
i. Daley v. LaCroix P claims emotional disturbance after she witnessed an electrical
explosion near her house.
a. Rule can have claim for ED when theres physical injury/ physical
consequences (after the fact) and not physical impact (injury at the time)
1. Measure is whether a normal individual would react w/ ED
2. Trying to limit liability b/c ED is easy to exaggerate/ fake, could lead to a
ton of cases. Cant let s recover for physical/mental harm that they are
worried about incurring in the future (consider DES daughters who
havent been diagnosed with cancer yet could sue for damages for not
wanting to marry, etc.TW says it can be serious ED without physical
injury)
ii. Thin skull doesnt apply in NIED

d) Bystander Rules
i. Witness child getting killedcan you recover?
ii. Dillion Rule (restatement and strong majority follow, got rid of zone of danger) - can recover
if:
a. Closely related to victim
b. Present at the scene of the event
c. As a result, suffers ED beyond whats anticipated
d. Do it flexibly on a case by case basis
iii. Zone of Danger Rule for contemporaneous observance (minority rule)
a. Have to be in zone of danger (ex: mother standing on curb watching her childs
accident couldnt recover, but sister might b/c stepped into the road at the time
of accident)
b. Have to be closely related
1. NY rule
iv. Most common rule requires to be (La Chusa case):
a. Closely related to victim
b. Present at scene and witness accident (Contemporaneous observance)
c. Suffers emotional distress as a result
d. Must be beyond the ED suffered by a disinterested bystander
v. Twerskis problem:
a. Very arbitrary way of deciding who can recover
b. Unfairly eliminates some s form recovering
e) Authors Dialogue 24 pg 423
i. T: I bet youll tell me that courts deny these claims because stand alone ED doesnt
represent loss in a broader societal sense
ii. J: any time a court denies recovery for what is valid claims I think something basic is at work
iii. T- then you dont buy the traditional explanation that courts are fearful that ED is faked?
iv. J- I do a bit. but a lot of these are real
a. Ed is important but maybe courts think the plaintiff is better to deal with it than
the courts
b. Ed cant be avoided, most causes of ED have nothing to do with tortious conduct
Page 42 of 60

v.
vi.
vii.

c. The def has accelerated the ED, but not prox caused it
d. Ed is always out there
e. Background risk like in Marshall v Nugent
Twhen parent is eyewitness to event we make an exception?
J-yes. That load of grief is unusual.
T- we dont make an exception when they dont witness it.


D.) Harm to Unborn Children and Parents
a) Usually cannot recover unless the fetus is considered viable (Werling)
i. Viable= able to live outside of the mothers womb
ii. Attempt to reconcile tort rules with the Roe holding
b) Wrongful life suits (Procanik case) doctor fails to tell parent that kid will be born with defect
i. Allows parents to recover for a child if born with injuries that parents would not have had to
pay for if they knew and selected to abort baby
a. He took away her choice to abort the baby
b. Can recover for the extraordinary med expenses related to defect (special
damages)
c. Cannot recover for pain and suffering (general damages--Cant evaluate living
with injuries verse not living at all)
c) Authors Dialogue 25 pg. 437
i. J- I dont get these wrongful life cases when courts say that life no matter what its better
than no life
ii. TWisnt that as good a reason to say no to these claims? The plaintiffs in these cases are
not suicidal. Its obvious they prefer to live so its weird when they insist they wish they
were dead.
iii. Jbig diff btw wishing you were never born, and having been born wanting to die. A jury
could certainly conclude in extreme cases that the plaintiff is so badly off that never being
born at all would have been better. What if its a short, painful life? Juries should be able to
find that such a life has a neg value.
iv. TWbut what about valuation?
v. J---Theyll figure it out
vi. TW---I just dont like giving legitimacy to the notion that someone would be better off dead

Chapter 7: Owner and Occupiers of Land
A.) Duties Owed to Entrants on the Land (3 Categories)

**Its the function of the court, not jury, to determine the status of the entrant
a) Ask yourself:
i. Are we in a category state?
ii. If not, have to get into reasonable care issue
iii. If yes, what category is he?
a. If licensee, is she a licensee throughout or has it changed?
b. If invitee, do B<PL
b) Trespasser
i. Owner only liable for wanton/willful conduct
ii. Dont have to make house/property safe
Page 43 of 60

iii.
iv.
v.

If you discover trespasser, you must warn of hidden dangers known to you
Obvious dangers, no liability
Child: special requirements
a. Liable to child trespassers if possessor of land knows that 1) children are likely
to trespass, 2) knows there will be an unreasonable risk of death/serious bodily
harm, 3) children are too young to discover the danger, 4)
maintaining/eliminating the danger is slight compared to risks, 5) possessor fails
to exercise reasonable care to protect children.
vi. Katko case

c) Licensee (social guests)
i. Duty only to warn about hidden dangers (same as trespasser)

d) Invitee (persons who are invited for business purposes/ persons who come onto land open to the
public)
i. May have to prevent danger in the first place
ii. Owed full duty of reasonable care
a. Duty to:
1. Discover danger
2. Warn of dangers
3. Repair dangers
iii. Once you have invitee, not asking limited duty question because you have full duty of
reasonable care
a. Once you have full duty, if danger is open and obvious is the P of B<PL

e) Gladon v Greater Cleveland was on subway platform, he was either attacked or slipped and fell.
He was hit by a train and seriously injured.
i. Rule court says once hes on the tracks, hes a trespasser (he wasnt permitted to be
there), so theres only a duty to warn of danger once hes discovered and train cant act
willful or wanton. No duty of full reasonable care.
ii. Twerski: the categories dont make any sense. Now that hes on the tracks, hes allowed a
lower standard of care doesnt seem fair
f) Hypos 48-49 pg 446
g) Carter v. Kinney
i. Bible study group, guest slips on driveway
ii. Court says he is a licensee
a. has duty to warn of known danger thats not obvious
b. Was this a known danger? No so wins
h) Keetons take:
i. Any time you have a limited duty rule, it could cut off the case at the outset
ii. On the other hand, B<PL is no set rule so do analysis

B.) Rejection of the Categories
a) Some jurisdictions do away with categories in favor of reasonable standard of care to all cases
i. NY and CA follow
ii. Restatement, Third adopts. ( 52 limits for flagrant trespassers duty just to not act
willfully/ wantonly; unless they are helpless/ imperiled, then duty of reasonable care)
Page 44 of 60

iii.

Rowland v. Christian was a social guest in s apartment. injured himself when a


cracked handle on the faucet broke and severed tendons in his hand.
a. Rule all persons owed reasonable standard of care on anothers property. Do
B < P*L to determine reasonableness under the circumstances
b. Instead of no duty q, q is now foreseeability OR what is the burden like to warn
b) Authors Dialogue 26 pg. 458
iii. TW: You critiqued Rowland and said that many courts wont agree with it. You were right
a. However, your critique is based on the reasoning that Rowland acknowledges
that, while the traditional categories no longer control, they remain relevant.
Many of the considerations that went into supporting the limited duties for
various categories of entrants to land are retained as matters to be considered
by juries. Thus, juries not only take into account the need of land owners to
utilize their property with considerable freedom, the lack of foreseeability that a
trespasser may be on the land, and the difficulty of imposing a duty to inspect
the property to discover to discover hidden risks, but they also should somehow
weigh the plaintiffs status, as such.
b. Asking jury to decide policy issues and they shouldnt
iv. Jim: I think thats right. Under common law categories, jury is given a rule, not common
law neg balancing test and asked to determine whether, adding in the s status in some
undefined way, it was unreasonable for her to undertake responsibility.
v. TW: You have it wrong. What the courts did in Rowland and Scurti were saying were not
addressed to juries at all. (Read page 459)
a) Twerskis take:
i. Under reasonable care duty, many more claims will get to a jury
ii. Violins will come out and liability will be imposed in cases that s should not be held liable
iii. Preferably to keep the meat cleaver around that can cut cases on no-duty

C.) Special Landowner Rules
a) Firefighter Rule
i. Original rule was to treat rescuers as licensees, but this resulted in limiting s from liability
in too many cases
ii. Now rescuers may recover if creates a risk that is outside the scope of the risks assumed in
their jobs
b) Recreational- Use Statutes
i. If a landowner opens up their land for free public use, then usually no duty is attached
(Provides partial immunity)

D.) Duties Owed to Those Outside the Premises
a) Traditional Rule:
i. Possessor of land is liable for harm negligently done on the premises that effects people off
the premises
a. BUT, possessor is not responsible for natural conditions on the land that cause
injury off the land
ii. Generally owe standard of reasonable care
a. Taylor v. Olsen rotten tree falls in road near owners premises and car got into
an accident because of it. If the tree were a known danger, then there might be
a duty to cut it down. But, putting burden of inspecting all trees for non-visible
decay is too high
Page 45 of 60

b. Q: Whether or not a should have known duty or if duty to urban vs rural


c. Just to reasonable care test THEN make should have known argument, which
is a factual question

E.) Duties Owed by Lessors
a) Common law rule was that lessor didnt owe duty to persons injured on land that had been leased.
Exceptions:
i. Vendor knew of the danger and concealed it to purchaser
ii. Premises leased for public use
iii. Premises retained under the landlords control (stairways)
iv. Negligent repairs
b) New rule: Lessors owe duty of reasonable care
i. Sargent v. Ross s 4 y/o daughter died when she fell from a stairwell in s building.
constructed the stairwell so that it was apparently too steep and the rails were too low.
a. Rule Landlords have reasonable standard of care. Do B < P L
i. Issue over who controls the stairwell. Twerski thinks its important
factor. WHO is supposed to act reasonably?

F.) Premise Liability: Securing Against Crime
a) In general, courts dont treat landlord as insurer against criminal acts of third parties
ii. However, they are liable for taking reasonable steps to minimize risks of criminal attacks
i. Posecai v. Wal-Mart Woman mugged at gunpoint in Sams Club parking lot, all her jewelry
is stolen.
a. Rule - Duty to prevent against crime when crime is foreseeable
i. 4 tests for determining foreseeability:
1. Specific harm: only aware if owner can foresee the specific harm
very restrictive (limited duty rule)
2. Prior similar incidents test: evidence of prior criminal conduct =
notice of future risk arbitrary results because its applied w/
different standards re: # of previous crimes needed to est. notice
(limited duty rule)
3. Totality of the circumstances: takes into consideration other factors
such as the nature, condition and location of the land and relevant
factual circumstances criticized as broad and imposing duty on
owners who have experienced any harm in the area (doing B < PL)
4. Balancing test (Applied here): addresses the interest of the business
owners and the customers. Focuses on foreseeability. (Again, doing
B < PL)
b. Issue: Even if duty is found, still a HUGE issue with causation
i. Can you prove that lack of the protection was the actual cause of injury
c. TW: Here, court was doing a B<PL analysis without calling it one so that is
doesnt go to the jury since there are policy issues.


Chapter 8: Affirmative Defenses
DEFINTION:
a) Defenses based on s conduct
Page 46 of 60

i.
ii.
iii.

may claim failed to act reasonably


may claim assumed the risk of injury
may claim didnt act in a way that would have lessened his injury

A.) Contributory Negligence
a) Used as a complete bar to recovery (old rule)
b) Butterfield v. Forrester had pole in front of his house (negligent), was riding his horse as fast
as he could and didnt see it (also negligent). injured and sues.
i. Rule barred from recovery b/c of contributory neg.
c) Courts tried to get around contributory neg. as a bar
i. Rarely directed verdicts
ii. Used last clear chance
a. If the finds the in a perilous situation (that they put themselves in) and does
nothing, then is not barred
iii. Contributory neg. not used as a defense for intentional torts or reckless/ wanton conduct
iv. Courts were very demanding in requiring that prove that the s negligence was both
cause in fact and prox cause

B.) Comparative Negligence
a) New doctrine that is used in most states (McIntyre)
b) Compares and by assessing their fault
c) Three ways to read comparative fault
i. Pure (NY Rule)
a. Assess s fault and reduce the damages accordingly ( 90% at fault, will recover
10% of damage award)
ii. Modified Comparative Fault (Most states adopt)
a. Greater than If s fault is not greater than s fault, then he can recover.
(i.e. if is 50% at fault or more). Once gets 51%, barred.
b. Less than If s fault is 49% or below, he can recover
1. Multi-parties: Court says will recover if his fault is less than the s %
of fault combined
d) Effect on Joint and Several Liability
i. Comparative negligence doesnt logically preclude joint and severable liability
a. J/S is a question of cause both people are 100% cause, but comparative
negligence talks about negligence and fault fault is not the same thing as
cause)
ii. McIntyre did something it wasnt asked to do; said comparative fault means you get rid of
joint and several liability
iii. Consider: gets into an accident with Big Sugar Daddy and Joe Shmoe (has 20K insurance
policy). Damages are awarded for $1 mil. Joe Shmoe is 70% at fault, BSD is 30%; if theres J/S
liability, BSD is going to foot the bill after Shmoe paid 20K.
a. Justification for J /S is 30% of fault is still 100% of cause.
b. Issue is if you dont hold them J/S then the P may get stuck with a bum who
cant pay and P is stuck assuming the cost of harm
c. However, you have s not paying their fair share b/c they have deep pockets (or
the other guy is indemnified by WC or low insurance policies)
d. Twerski thinks getting rid of J/S liability with threshold % might be a good idea.
Not logically imperative that you get rid of J/S.
Page 47 of 60

a. Ranson v Kitner
i. Trespass to chattels
ii. Intentional tort doesnt mean there shouldnt be comparative
fault

e) What Counts as Fault Restatement, Third 8
a. Facts for Assigning Shares of Liability:
1. The nature of the persons risk-creating conduct, any awareness or
indifference with respect to the risks and harm created
2. The strength of the causal connection between the persons risk-
creating conduct and the harm
i. Twerski (2) what does this mean? must be 100% cause in
fact. They must be talking about prox cause here (take into
account scope of the risks)
b. To reduce recovery, s fault must be the cause-in-fact and the proximate cause
of her own harm
c. HYPO 50 pg 490


C.) Special Problems with Comparative Fault
a) Last Clear Chance (dead)
i. Can be considered by the jury as a factor in deciding % of fault, but its no longer an
independent doctrine to negate s contributory negligence.
b) Comparative Negligence Meets J/S Liability
i. McIntyre declared that J/S was no longer necessary when the court used comparative fault.
a. Other courts disagree
b. this means you are only ever responsible for your % of the fault (not fair
because you are 100% at cause)
c) Comparative Negligence as a Defense to Intentional or Reckless Conduct
i. Most courts allow the jury to assess the relative fault of the and the reckless
ii. When comparing s negligent conduct w/ s intentional tortious conduct, many courts
wont reduce s recovery based on her own fault
a. Might be exception if provoked s intentional conduct
d) Rape Cases and Comparative Fault
i. Cases where rape victim sues a third party for inadequate security, some will say that
person being raped failed to take precautions that would have prevented the rape
e) Allocating Fault Among Multiple Parties
i. Problems come up under modified comparative fault when is suing multiple parties (who
are individually less at fault than , but more at fault if aggregated together)
a. Majority will allow to recover if s fault is less than the combined fault of the
s.
i. This is FAULT not CAUSE, both are the cause
ii. HYPOS in class notes
f) Interplay Between Comparative Negligence and Prox Cause
i. With comparative fault in play, prox cause doesnt get many directed verdicts.
ii. Prox cause is still an issue, but its much weaker than before
b. Real question is if was in scope of the risk comparative fault
goes to jury
Page 48 of 60

g) Rescue and Comparative fault


i. Many courts wont reduce recovery for rescuers unless their conduct was grossly negligent
or willful/ wanton
h) With Comparative Fault, Crime Doesnt Pay (at least sometimes)
i. Barker/Manning Rule no duty when actor engages in illegal activity someone who breaks
into your house slips and falls robber cant sue for negligence (public policy)
ii. Alami v. Volkswagen drunk driver sues VW saying his car wasnt adequately crashworthy.
Sues for the add-on injuries, not for the crash. Concedes there was a serious violation of
the law going on
a. Rule - Court says Barker/Manning rule doesnt bar recovery, but still going to do
comparative fault
b. B/M rule didnt apply here because VW still has a duty to make safe cares, the
duty didnt arise out of the illegal act. (TWwho cares how the crash happens)
c. Twerski VW is responsible for making a safe car that will hold up in a crash. It
doesnt matter what led to the crash (criminal activity). They still have a duty.
But I would apply the Barker/Manning rule.
iii. Barker/Manning rule might not be needed to bar recovery. In states w/ modified
comparative fault, might still get nothing since juries are likely to say that drunk drivers
are more than 51% at fault

D.) Assumption of Risk
a) Traditional rule: if voluntarily assumed a known risk, he was barred from recovery
i. Tried to get rid of AOR as complete bar when using comparative fault
b) Express Assumption of the Risk
i. agreed to allow the to be exposed to her conduct only if the agreed to exculpate the
from liability for negligence
a. If agreement is valid, then no comparative negligence
ii. Exculpatory agreements are ok when:
a. Clearly spells out parties intentions
b. Nothing in the social relationship between the parties weighing against
enforcement
c. Not against public policy
iii. Public Policy exception
b. Courts can reject agreements when they deem the agreements have
detrimental effect on public policy
iv. Stelluti woman fell off a spin bike when the handlebars fell off. Had signed a contract
agreement w/ gym that day exculpating them from liability.
a. Rule Contract was enforceable, was barred from recovery. No public policy
issues at play, this is a gym where ppl know the risks.

c) Implied Assumption of the risk
i. HYPOS pg 511
ii. In order to get to AOR defense, must first show that the breached a duty and there was
negligence.
iii. Rule: Dont get to affirmative defenses until theres a prima facie case (duty/ breach/
negligence)!

Page 49 of 60

iv.

v.

vi.

iii.

iv.

v.

IAOR defense: If a participant makes an informed estimate of the risks involved in the
activity and willingly undertakes them, there can be no liability if he is injured as a result of
those risks
Fleming Critique:
a. You dont need AOR b/c the case will either be covered by no-duty rule or
comparative fault
b. When you use AOR, it gives you the wrong result
1. Ex: person playing catch near a pool
i. Situation 1= owner of land has no duty to warn
a) is not negligent for anything done by
ii. Situation 2= duty to warn therefore is negligent
a) BUT if acted unreasonably, then comparative fault will
find him responsible
b) If acted reasonably, then he should recover
c) Under AOR he would still be barred, which is incorrect
Blackburn Gets rid of assumption of risk principles, use comparative fault instead.
a. Primary vs. Secondary assumption of the risk
1. Primary
i. has no duty to protect because is under impression
knows of risk and accepts it
ii. Serves as complete bar
2. Secondary
i. owes duty of care but knowingly encounters risk posed by
s breached duty
2. TWProblem isuse which form of comparative fault? No reason for
it.
Authors Dialogue 27 pg 516
a. TW: You agree with Blackburn that AOR as an independent doctrine should be
laid to rest. HYPOwhat if Jack gets into car with Giant knowing he is drunk?
b. Jim: I wouldnt give Jack a nickel. Giant has no duty to Jack if Jack agreed.
c. TW: Thats an awkward way to talk about duty. Giant has breached a duty to act
reasonably to sleeping Jill. If he has no duty to Jack, its not because of some
unspoken contract, its because Jack voluntarily decided to encounter a risk.
d. Jim: No. Giant gets off on no duty grounds based on their tacit agreement.
e. TW: Jack did act voluntarily. Even if Jacks car broke down and it was freezing,
Giant didnt put him in the dilemma, the car did. Assumption of risk works fine.
f. Wendel: It feels strained to talk in no duty rules
Turcotte v. Fell
a. Horse jockey that fell and tried to sue other horse owner and venue owner
b. Cant do it under AOR so do it under duty.
c. Risks inherent in the activity.
d. TWwhat do I do with amateur players?
1. Do classic voluntary AOR analysis for no duty decision.
2. Twerski law review article
Authors Dialogue 20 pg 520
a. Could do scope of risk analysis asking if you should have fence for risk takers
who want to dive in the pool?
1. Are risk takers foreseeable?
Page 50 of 60

2. You can TRY to do scope of risk analysis but it comes down to AOR
i. Diver knew the risk, wanted that exact risk
ii. In NY/FL you have to do it backward. You assumed the risk do
no duty.

vii. Twerskis Take on AOR:
a. You need AOR argument for certain cases (cant always talk in no-duty terms)
b. You can say has no duty to people who seek out risks (ex: athlete choosing to
run towards a pool to catch a ball) or people who accept risk (Jack knowingly
gets into the car w/ drunk Giant), but it just doesnt sound right because they do
still have a duty
c. Long live assumption of the risk! just calling it a different name
ii. TW: There are cases where you voluntarily take risks (fly ball)
but that doesnt make you negligent
iii. There ARE cases where AOR should apply (Turcotte)
1. Strong argument they shouldnt recover
b. Though, Jim says if you keep AOR, courts are going to mess it up (and Twerski
says this argument has merit)
1. Jim: does not have duty to those who seek out risk

E.) Avoidable Consequences
a) has duty to take reasonable steps to alleviate her injuries
i. Ex: s injured by s negligence. She unreasonably refuses surgery and suffers permanent
damage that she wouldnt have had she accepted. is not responsible for the add-on
injuries.
a. Issue of causation
b) Novko v. State of NY Dairy farmer is in an accident. Afterwards, he takes steps to lessen the injury.
Court says he should have changed his profession (be a salesman instead of a farmer in order to
mitigate damages/ injury)
i. Rule its unreasonable to hold that should change his profession, he took substantial
steps to lessen injury, so he shouldnt be barred from recovery
c) has burden of proof that s post-injury conduct increased his own harm, and by how much
d) Seat Belt defense
i. Most jurisdictions wont allow to use evidence that failed to use seat belt and then got
into an accident as avoidable consequences defense b/c its a preventative measure and
not after-the-fact aggravation of injury
a. Some states do apply (NJ/NY)

F.) Non-Conduct-Based Defenses
a) Immunities
i. Most family immunities have been eliminated
ii. Charities are no longer immune because they are now big business
iii. Government still somewhat immune
iv. Tippett v. US
C. Yellowstone sheriff told s they can pass moose but moose attacked them
D. District court dismissed case because they were barred by discretionary function
exception of the ACT (affirmed)
v. Military actions immune if incident to service
Page 51 of 60

a. What is incident to service = subject of debate


State immunity similar to government, but with more pockets of immunity
Municipal immunity = regulated by state
Riss v. City of New York
C. Rejected suitor threatened to kill and she went to police but they ignored it
D. Police cant be sued for not protecting the public
ix. HYPO 51 pg. 542
x. Sometimes cant sue the cops if there is a special relationship between you and the police
(such as you inform them, and agree to testify about the person you told on)

b) Statute of Limitations
i. Statute of limitations starts in most states were knows or reasonable should know that
she's suffered injury
ii. In sexual abuse cases, most states toll the statute of lim for minors until they reach the age
of maturity
c) Statute of Repose
i. Some states have enacted legislative statutes of repose to bar claims after a certain period
of time. They protect s from litigation far past the injury.

Chapter 9: Joint Tortfeasors

A.) Joint Tortfeasors
a) Three situations in which each tortfeasor will be responsible for 100% of s damages even though
others are involved:
i. s act in concert to cause harm (Ex: drag racing)
ii. s are held liable by operation of the law (ex: respondeat superior)
iii. s cause a single indivisible harm (ex: two hunters shoot, one hits left eye, the other hits R
eye- blindness is indivisible harm)

b) Concerted Action
i. is held liable for the acts of others b/c he expressly or impliedly agreed to engage in
activity with ppl who turn out to be tortfeasors
ii. Herman v. Westgate Ppl are on a boat for a party. A group starts to fool around and throw
ppl off the boat. is badly injured. All held liable if they participated in the act, doesnt
matter that they didnt do the actual throwing.
iii. Restatement 876. Persons acting in Concert. Subject to liability if:
a. Does tortious act in concert with other or pursuant to a common design
b. Knows the others conduct is a breach of duty and give substantial assistance or
encouragement (standing next to someone beating and saying give it to him)
c. Give substantial assistance to the other in accomplishing a tortious result, and
separately considered constitutes a breach of duty
1. Hard to determine what substantial assistance is (Podias case two
cars get into accident, Podias is left on the road seriously injured. Other
kids left and didnt call for help. Podias was then run over by a second
car. Although not all actors had an affirmative duty to aid, they gave
substantial assistance not to report)
i. By and large, substantial assistance is a jury Q

vi.
vii.
viii.

Page 52 of 60

c) Liability by Operation of Law


i. Ex: when respondeat superior applies, even if only the employee acts negligently, the
employer is a joint tortfeasor with employee,
a. Each bears enter responsibility for s injuries

d) Indivisible Injury
i. Both s are negligent and cant divide up who caused what both are responsible 100% for
injury
ii. American Motorcycle - kid enters motorcycle race, gets badly injured. Sues companies that
put on the race. s say parents and kid are both negligent for entering the race.
a. Comparative fault doesnt get rid of J/S liability
b. The fact that one of the torfeasors is immune from suit doesn't relieve another
tortfeasor of his liability for damage which he proximately caused.
c. Goes opposite of Mcintyre
d. let them recover and let s work it out
e. why should have no recovery just because poor is more at fault?

e) Joint and Several Liability
i. Now, many jurisdictions have gotten rid of J/S liability
ii. Some have modified
a. Some apply J/S liability for economic harm only (not for pain/suffering, punitive
damages NY rule)
4. Case with 200,000 in econ and 800,000 in pain, the 200,000 is J/S
b. Some apply J/S liability after a threshold % has been met
c. 2 states apply J/S only when is faultless
d. Hybrid based on type of damages (NY)
1. Economic loss= joint and several
2. Non-economic loss= threshold question
e. States are divided but most say when one Ds conduct is intentional, the
negligent P will pay only a small amount of money
iii. Restatement, Third, Apportionment of Liability
i. A person who is liable to another based on a failure to protect the other from a specific
risk of an intentional tort is J/S liable for the share of comparative responsibility
assigned to the intentional tortfeasor in addition to the share of comparative
responsibility assigned to the person.
1. Rape casewill you allow security to pay everything? Restatement says no.
iv. Problems with elimination of J & S
a. May rob of chance of recovery
b. If a deep pocket defendant is only held to be a low % responsible and the other
is insolvent, then is going to end up eating the cost


f) Satisfaction of a Judgment and Aftermath: Contribution and Indemnity
i. P is only allowed one full recovery (any amount paid by 1 D is deducted from the recovery
sought from the 2nd D)
ii. Contribution
a. D whos paid more than his fair share, can seek contribution ($) from other Ds
b. No reason for contribution in jurisdictions that have gotten rid of J/S liability
Page 53 of 60

1. D only pays his proportional share of the total damages


Indemnity
a. In certain cases, D who either settled or paid a judgment may be entitled to full
reimbursement from joint tortfeasor
b. D looks to recover from the immune tortfeasor that he ended up paying for (ex:
Employer seeking to recover from negligent employee

g) Settlement and Release
i. At common law, settlement w/ one D released all joint tortfeasors from liability
a. However, discouraged settlements b/c P felt like they were deprived of their
rights against all parties
ii. Settlement and Contribution
a. If contribution is allowed against the settled D, they would never want to settle
(not buying them peace, just delaying a lawsuit)
b. However, if contribution isnt allowed, sweetheart settlements btw friendly
parties will leave the other non-settling Ds footing the bill
c. Some jurisdictions (NY rule) release Ds from the percentage of their fault (thats
ultimately determined at trial) if they settle.
1. If Ds fault % is higher than P thought it would be, the difference comes
out of Ps pocket
d. Twerski: this is a problem with no good solution. Release of % of fault is not
terrible (but not good) Full contribution and no contribution are terrible.

h) When the Issue of Divisibility of Damages is Unclear
i. Courts should try to apportion fault as best they can, but sometimes its difficult
ii. Courts have decided to hold all of them liable, then let them try to prove themselves out
a. Michie v. Great Lakes P sue 3Ds for releasing pollutants into the air. No way to
tell who caused what damage.
1. Rule if jury cant apportion fault, then Ds are treated as joint
tortfeasors and J/S liability applies
2. ** Fault is not the same as cause
3. Burden shifts to Ds to prove that they only caused X amount
iii.



Chapter 10: Strict Liability
A.) Strict Liability
DEFINITION
a) Liability w/o fault, based only on the fact that s lawful conduct caused harm to the
i. is liable even if he exercised reasonable care and doesnt intend to interfere w/
ii. Still subject to limitations not absolute liability

b) Possession of Animals
i. Livestock
a. Owners are strictly liable
ii. Wild animals
a. Owners are strictly liable (need to prove D owned the animal)
iii. Domesticated animals
Page 54 of 60

a. Owners are liable only if P proves that owner knew that the animal was prone to
violence
b. Courts dont treat entire categories of animals as having vicious propensities
(ex: breeding bulls Bard case)
1. Have to show that D was aware that the particular animal was prone to
violence (one bite rule)
2. Courts wont apply regular negligence standard b/c these cases have
special rules (Bard)
c. Some states have gotten rid of distinctions between livestock and domestic
animals (all owners of animals need to be aware of the animals dangerous in
order to be held strictly liable)

c) Abnormally Dangerous Activities
i. Rationale for strict liability
a. Used in situations in which reasonable care will not prevent accidents/harm
(abnormally dangerous activities)
b. In these situations, it is impractical to ask/evaluate the negligence question
ii. Conditions where liability should be imposed (Restatement 20 Abnormally Dangerous
Activities)
a. Risk of harm is great
b. Harm cannot be prevented by exercise of due care (dynamite/ blasting)
c. Activity is not a matter of common usage
d. Activity is inappropriate in the place it took place (ballooning in NYC)
e. Value to the community does not outweigh the unavoidable risks
f. D brings something unnatural onto his land and it causes damage to neighbors
prop (Reynolds/ Fletcher)
iii. Scope
a. Ds activity must still be the proximate cause of Ds injury
b. Liability is only imposed if injury results from the kind of risk created by Ds
conduct
1. Foster - Drilling and blasting is dangerous but cannot hold D responsible
for killing Ps minks which were excited 2 miles away
i. Not foreseeable (killing of minks is not directly related to the
risk posed by blasting)
iv. Attempts to limit SL
a. Indiana Harbor American Cyanamid makes toxic chemical. N.A. corp leases RR
tank Missouri Pacific RR co goes through IL IN Harbor is the small
switching line. Tank leaks chemicals, costs close to $1 mill to clean up. Should
the shipper of the chemical be held SL? The manufacturer?
1. If negligence doctrine can be applied, it should be
2. Do not want to overuse SL, should be limited to certain situations
3. Places burden on the transporters not the manufacturers
4. Transporters know more about safety and best routes, to place the
burden on the manufacturer would be counterproductive and unfair
i. Most courts wont hold manufacturers of dangerous chemicals
liable under idea of abnormally dangerous activity
ii. Twerski: disagrees with Posner for several reasons. 1) The
transporter/ switching line doesnt know whats in the tank and
Page 55 of 60

v.

should be obligated to find out; thus theres no reason why they


should figure out the safest route. 2) Shouldnt deal with this
case by only relying on negligence. Doesnt think you can avoid
some derailments.
iii. Twerski: Strict liability depends on defining the category of
activity as needing the special liability rule. Dont do a case-by-
case analysis. Either shipping of toxics is or is not subject to SL

Resilience of Negligence Principle
a. Courts rely primarily on negligence, not strict liability when they decide
abnormally dangerous activity cases
b. One reason: SL calls for a system of social insurance that is not workable


Chapter 11: Products Liability
A.) Before the Products Liability Revolution (1960s)
a. Two ways to bring case:
i. Negligence COA
1. No privity required
2. Company didnt live up to disclaimer
3. Had to prove manufacturer at fault
4. Statute of limitations started at time of injury
ii. Contract COA (implied warranty of merchantibility)
1. Had to be in privity (consumer down the chain cant sue manufacturer)
2. Strict liability COA (didnt need to prove fault)
3. Had to prove product was defective at time of sale and defect caused the harm
4. 4 year statute of limitations from time of sale

B.) New Tort for Products Liability in 1965 : Restatement, 2nd 402 (Prosser)
a. Privity free (can hold anyone in the chain accountable)
b. Anyone who sells a defective product which is unreasonably dangerous to the user can be
liable
i. Has to be defective when sold
ii. Doesnt matter if seller acted reasonably (due care) Strict Liability COA
iii. Comment i : unreasonably dangerous means dangerous beyond what the ordinary
consumer thinks of (i.e. whiskey isnt dangerous b/c it can make people drunk)
1. ** Written with manufacturing defects in mind
2. Problems came up with defining defect (what do you do with design defects
and failure to warn?)

C.) Products Liability Restatement (adopted in 1998, written by Twerski and Henderson)
a. Different from 402A in that it breaks the single defect category into 3 parts:
i. Manufacturing Defect
ii. Design Defect
iii. Failure to give adequate warning

D.) Manufacturing Defect
Page 56 of 60

a. Product doesnt meet companys own standard


i. Have to prove defective at time of the sale (P didnt do it later)
1. Most often use expert testimony
2. Can make res ipsa inference more probably than not accident occurred
b/c of product defect
a. Welge v. Planters guy goes to sprinkle peanuts on his sundae and the
bottle just breaks in his hand. D has burden to show the defect didnt
happen in their hands. P doesnt have to rule out every possibility can
argue more probably than not

E.) Design Defect
a. Argument that product should have been designed better or not at all (courts hesitant to say
dont make at all)
b. Attacked the whole line, not just one product
c. Imposes strict liability to whole chain of distribution
d. Standards for determining:
i. B< P L / Risk Utility Balancing Reasonably Alternative Design (RAD)
1. Once P proposes a RAD, burden shifts to D to refute
2. Some courts have said RAD is just a factor and not a necessity to proving design
defect
a. Vatour leg press machine injures P, there was no other place to put
the safety stops.... Court worries that coming up with RAD puts too high
of a burden on P
b. Twerksi disagrees. RAD IS NOT JUST A FACTOR TO DESIGN DEFECT; ITS
THE ANSWER TO R/U ANALYSIS!
3. By proving a RAD exists, you show that someone made a bad risk utility decision
at some point in time

ii. Consumer expectation test
1. Impose liability if the product disappoints consumer expectations (dont need to
come up with RAD)
2. PROBLEMS:
a. Basically making a res ipsa inference, dont need to call it consumer
expectation
b. CET negates the open and obvious danger test just b/c the product
is obviously dangerous (consumers expect it to be), doesnt mean the
product is safe or that the manufacturer shouldnt still be held liable
c. R/U analysis might conflict with what consumers expect
i. R/U should win
d. CET gets rid of causation Q
3. Twerskis overall take: CET isnt a legitimate test b/c there are no answers to the
problems. Should just be doing R/U analysis for design defect cases (takes into
account reasonable person, dont need to change it to reasonable consumer).

F.) Two-Prong Test for Defect
a. Do CET, but if it fails, then do B < P L (Soule Ps car crashed, said design defect in car caused
her ankles to get crushed. CET doesnt help here b/c consumers dont know enough about
crashworthiness of cars)
Page 57 of 60

b. Soule if parties disagree which test to use (CET vs. R/U), then jury gets a R/U instruction
i. Twerski parties are never going to agree to use CET b/c its either going to help one
side of the other. Ultimately just concluded to use B < P L for defect cases

G.) Category Liability -
i. Some products have no RAD and fail B < P L b/c they are so dangerous and have such
low societal utility
ii. Courts will say an entire category of products are defective in VERY RARE INSTANCES
1. OBrien guy dives into an above ground swimming pool and gets injured; says
the pool was defectively designed with vinyl. P doesnt come up with a RAD. Not
going to say that above ground swimming pools are so dangerous that their
risks outweigh any utility and to get rid of whole line.
2. Not going to do it for cigarettes either (Adamo v. B&W Tobacco)
3. Restatement examples: exploding cigars, toy guns that shoot rubber pellets
4. Think legislature should be making these decisions and not courts

H.) Failure to Warn
a. Says manufacturer should have given more info about the risks of the product (begs the Q how
much more would have been enough)
i. Every P is going to say you should have warned about what happened to me
b. Two types of warnings:
i. Warnings that reduce risks (dont use blow dryer near water)
ii. Warnings of non-reducable risks (this drug has 1/100 chance of causing cancer; goes to
informed consent issue)
c. Adequacy of the warning is always a Q for the jury

d. Still have to prove causation
i. If there was a warning, would P have heeded to it?
1. Most jurisdictions will give P the benefit of the doubt and assume he would
have heeded to the warning
2. Hard for D to rebut (though still ways to defeat the presumption)
e. D has to have knowledge of risks for liability
i. Show that D knew or should have known
1. Basically a negligence test
ii. Manufactures dont have a duty to warn about risks that are unknown to the industry
(state of the art defense)

f. Still have to show proximate cause
i. P must prove defect was the proximate cause of her injury (w/i the scope of the risk)
ii. Courts are fuzzy on the rules of when prox cause gets taken from the jury and decided
as a matter of law
iii. Union Pump Pump defect cause it to catch fire. P puts out the fire. Decides to take the
shorter, but more dangerous route back instead of the safer one, slips and falls. Court
finds Ps injury was not w/i the scope of the risks est. by Ds defective pump

g. Comparative Fault
i. Even if you get past prima facie case of product defect, damages can be reduced if P is
also at fault
Page 58 of 60

ii. P cant act completely stupid and then claim strict liability for product defect

Chapter 12: Trespass to Land and Nuisance
A.) Trespass to Land
a. If you're on another's property you're liable for trespass
b. Dont have to show land was harmed, you're just allowed to have exclusive right to your prop
c. Doesn't matter that the trespasser thought he was reasonably on the land (i.e. thought it was
his own)
d. Not liable person didn't intend to be the land (like he lost control of his car or something)
e. Remedies: nominal damages for minor intrusions, rental value for use of the land, damages from
D's conduct while trespassing, or injunction to get off the land

B.) Nuisance
a. Protects a person's right to the use and enjoyment of their prop
b. Intangible intrusions to land (noise, pollution)
c. Fairly flexible idea (involves cost/ benefit analysis)
i. Most courts draw a distinct line between the two torts. Though in pollution cases
sometimes the line is blurred (Bradley)
d. Private Nuisance:
i. P needs:
1. basis for liability
a. Still have to show that D's conduct was tortious (either it was
intentional, negligent, or subject to strict liability)
2. Significant harm
a. Can't be just an annoyance that happens when living in an urban society
(church bells during daytime, dust, etc.)
3. And unreasonable invasion on P's land
a. harm is severe enough that its unfair not to compensate the P for it
b. based on B < P L analysis
c. Can consider whether P was there first, i.e. he bought the land and then
D initiated nuisance (though not all courts will say this is a factor)

e. Public Nuisance:
i. an unreasonable interference with a right common to the general public
ii. P needs to show:
1. D's conduct involves a significant interference with public health, public safety
or public convenience
2. P has to suffer harm that's different from the general public
iii. Damages:
1. Can be injunctive or compensatory
2. Some courts say they are not going to impose injunctions if the injurer's utility is
high (Boomer case - not going to impose $345 investment in research and put
300 people out of work for a cement plant to stop operating or try to find a less
polluting alterative)
iv. Coase Theory on Nuisance:
1. Shouldn't ask who is "at fault," but rather whose use of land should be
preferred
Page 59 of 60

a. Decide which competing interest should win. Example: pollution creates


negative effects to neighboring landowners, but is a side effect of
company's productivity see how much one side would give/ take to
eliminate the nuisance or keep it
b. Problem could be resolved by bargaining to an efficient outcome.
c. When transaction costs prevent parties from negotiating successfully,
the law should step in



Chapter 13: Damages
A.) Punitive Damages
a. Used for retribution (punish the D for egregious wrongdoing), compensation and deterrence
(prevent D and others from acting this way)
b. Only awarded in conjunction with compensatory damages
c. Conduct required differs by jurisdiction
i. Some require gross negligence, wanton disregard, or actual harm

d. Determining the amount of the award
i. Juries have little guidance and will sometimes give huge awards
ii. BMW v. Gore standard:
1. Degree of reprehensibility
2. Ratio between punitive damages and actual harm/compensatory damages
3. Difference between award and criminal/civil penalties based on similar cases
iii. State Farm v. Cambell
1. looks at Gore standards and redefine the ratio to a single digit (i.e. 1:9 or
below)
2. Concludes that out-of-state evidence and dissimilar acts (Ds wrong conduct in
general) cannot be included in reprehensibility analysis
iv. Phillip Morris v. Williams returned a verdict w/ 100:1 ratio of punitive damages:
compensatory
1. Rule wrongs to other people besides P can be considered in deciding
reprehensibility, but it should not be considered when jury assesses damage
about
a. D shouldnt be punished for harming non-parties

e. Questions that are going to come up:
i. Ratio is still an issue
1. Three justices dissented Phillip Morris Thomas, Scalia, Ginsburg
a. Thomas/Scalia say there is no constitutional ban on excessive punitive
awards
b. Ginsburg says this isnt an issue for the judicial system; caps should be
set by legislatures
ii. Constitutionality
1. How many times can you punish the same D? Multiple punitive awards?
a. Due process claim: depriving D of prop when he has already been
punished for the same crime (double jeopardy?)

Page 60 of 60

You might also like